Critical Care/Emergency Medicine Flashcards

Page 252 - Page 329

1
Q

A 42-year-old woman sustained a motor vehicle accident (MVA) 2 days ago and was brought to the Emergency Department by ambulance. She was resuscitated accordingly, and was admitted to the hospital due to fractures of ribs and her left humerus. Today, she is found to be in respiratory distress. Pulse oxymetry shows oxygen saturation of 89% on room air. She has a blood pressure of 130/85 mmHg, pulse rate of 100bpm in both arms, respiratory rate of 30 breaths per minute, and a temperature of 36.7°C. Lungs and heart are clear on auscultation. Supplemental oxygen by facial mask is started, and a chest X-ray obtained that shows whitening of the left pleural angle. Which one of the following is the most likely diagnosis?

A. Traumatic rupture of the aorta.
B. Pulmonary contusion.
C. Pneumonia.
D. Hemothorax.
E. Atelectasis.

A

B. Pulmonary contusion.

Except for traumatic rupture of the aorta, all the given options have respiratory distress and hypoxia as a common and early presentation. Furthermore, equal pulses of both arms make traumatic rupture of the aorta a very remote possibility.

With atelectasis (option E) and post-obstructive pneumonia (option C), fever is expected to be present. This patient is afebrile making these two less likely.

In hemothorax (option D), chest exam is not normal, and the following are present:
* Mild to moderate shortness of breath (commonly)
* Absent breath sounds on the base of the affected side
* The base of the affected lung is dull to percussion
* Faint and distant breath sounds on the apex of the affected side

With respiratory distress and hypoxia following blunt chest trauma 24 - 48 hours after the incident and a normal chest exam, pulmonary contusion would be the most likely diagnosis.

Pulmonary contusion is the bruising of lung parenchyma due to trauma. It is most commonly caused by direct blunt trauma to the chest wall, or in explosions, or a shockwave associated with penetrating trauma.

The condition may not show up immediately after the injury and becomes evident 1 or 2 days after the trauma. This necessitates close monitoring of every patient with considerable trauma to the chest wall. Hypoxia and respiratory distress are main manifestations.

Pulmonary contusion is very difficult to diagnose only based on chest exam because exam findings are almost always inconclusive.

Chest X-ray is the initial diagnostic tool; however, chest X-ray often underestimates the size of contusion and tends to lag behind the clinical picture. In pulmonary contusion, X-ray shows whitening of the affected area(s).

In one-third of the patients, radiological characteristics may take an average 6 hours to develop. The true extent of injury takes 24-48 hours to develop. When the radiologic appearance is evident in a short time after the incidence, a CT scan must be performed for assessment of associated injuries.

Pulmonary contusions usually resolve in 3 to 5 days, provided no secondary insult occurs. The main complications of pulmonary contusion are acute respiratory distress syndrome (ARDS) and pneumonia. Approximately 50% of patients with pulmonary contusion develop ARDS. This percentage increases to 80% if more than 20% of the lung is affected. Direct lung trauma, alveolar hypoxia, and blood in the alveolar space are all major activators of an inflammatory pathways resulting in acute lung injury.

Pneumonia is also a common complication of pulmonary contusion. Blood in alveolar spaces provides an excellent growth medium for bacteria. Clearance of secretions is decreased with pulmonary contusion, and this is augmented by any chest wall injury and mechanical ventilation. Good tracheal toilet and pulmonary care is essential to minimize the incidence of pneumonia in this susceptible group.

NOTE - Ruptured aorta (option A) presents with hemodynamic instability (hypotension and tachycardia) and widened mediastinum on chest X-ray.

How well did you know this?
1
Not at all
2
3
4
5
Perfectly
2
Q

A 24-year-old man is brought to the Emergency Department after he sustained a roadside accident. On examination, he is conscious and fully alert, has a blood pressure of 135/90 mmHg, pulse rate of 100 bpm, and respiratory rate of 22 breaths per minute. There are several tender bruises over his chest. A part of the chest wall on the right side has paradoxical respiratory movements. He is started on opiate analgesics and supplemental oxygen by face mask. A few hours later he develops drowsiness and severe dyspnea. Bedside pulse oximetry shows an oxygen saturation of 87%. Which of the following is the most appropriate management for this patient?

A. Tracheostomy
B. Intubation and ventilation.
C. Strapping the chest.
D. Tracheostomy.
E. Cricothyroidotomy.

A

B. Intubation and ventilation.

The clinical picture suggests flail chest. Flail chest occurs when 3 or more adjacent ribs are each fractured in 2 places, creating a floating segment comprised of several rib sections and the soft tissues between them. This unstable section of chest wall moves in the opposite direction of the uninjured, normal-functioning chest wall (paradoxical movement), and is associated with significant morbidity from pulmonary contusion. Abnormal motion can be difficult to detect and make the diagnosis difficult.

Initial management of flail chest includes oxygen and close monitoring for early signs of respiratory compromise, ideally using both pulse oximetry and clinical observation.

Pain management and pulmonary toilet is mainstay of therapy in patients whose respiration is not yet compromised. Options for pain control include patient-controlled analgesia (PCA), oral pain medications and indwelling epidural catheters.

Respiratory compromise, if develops, is a result of underlying pulmonary contusion. In alert, otherwise uncompromised patients, continuous positive airway pressure (CPAP) may spare them from the need for intubation and mechanical ventilation. However, intubation and mechanical ventilatory support is required in:
* Patients with severe injuries
* Patients with respiratory distress
* Progressively worsening respiratory functions
* Patients with respiratory suppression because of excessive narcotic pain control

NOTE - Intubation and mechanical ventilation should be followed by prophylactic chest tube insertion (often bilateral), because the ragged edges of broken ribs my lead to pneumothorax.

This patient has developed respiratory distress despite being on analgesics and oxygen due to either respiratory suppression from analgesia or underlying pulmonary contusion. Therefore, he should be intubated and mechanically ventilated. This then should be followed by prophylactic chest tube insertion.

Stabilization of the segment with manual or object pressure restricts chest wall expansion and interferes with appropriate respiratory mechanics and is no longer used.

Other options have no role in management of flail chest and its complications.

How well did you know this?
1
Not at all
2
3
4
5
Perfectly
3
Q

A 34-year-old gang member sustains a penetrating chest trauma and is brought to the Emergency Department. On examination, his vital signs are stable. He has no shortness of breath. A wound is noted in the right hemithorax. Initial evaluation is consistent with an open chest wound without any air-sucking or bubbling. Auscultation reveals decreased breath sound in the right lower lung field, and percussion is associated with a dull note. Which one of the following would be the most appropriate next step in management?

A. Strap the wound with pressure.
B. Underwater-sealed chest tube.
C. Needle aspiration.
D. Intubation.
E. Thoracotomy.

A

B. Underwater-sealed chest tube.

The main concern in the scenario is possible hemothorax evident by decreased breath sounds and dullness on percussion. Water-sealed thoracostomy drainage is the primary mode of treatment for hemothorax.

Injuries leading to massive hemothorax include aortic rupture, myocardial rupture, and injuries to hilar structures. Other causes include injuries to the lung parenchyma and intercostal or mammary blood vessels. A volume of 30 mL is needed for hemothorax to manifest on an upright chest X-ray. In skilled hands, ultrasound can diagnose hemothorax accurately.

Although the chest wound seems open on exam, there is no sign or symptom suggestive of a sucking open chest wound potentiating tension pneumothorax for now. The main concern with open chest wounds is the fact that air tends to enter the pleural cavity through the wound, rather than into the lung through the trachea once the wound diameter exceeds 75% of the diameter of the trachea during inspiration. This will quickly lead to pneumothorax and tension pneumothorax.

NOTE - Immediate management of a sucking chest wound is with a three-side-sealed wound covering to produce a one-way valve mechanism

Option A: Wound strapping is not an option for management of hemothorax.

Option C: Immediate decompression with needle aspiration is used for urgent management of tension pneumothorax.

Option D: Intubation is not an option for management of patient with hemothorax.

Option E: Thoracotomy is the procedure of choice for surgical exploration in the following situations:
* An immediate bloody drainage of ≥20 mL/kg (approximately 1500 mL)
* Persistent bleeding (generally >3 mL/kg/hour)
* Shock despite initial treatment

How well did you know this?
1
Not at all
2
3
4
5
Perfectly
4
Q

A 2-year-old Aboriginal boy has ingested 2 lithium batteries 2 hours ago, and is brought to the Emergency Department. Chest and abdominal X-rays confirms the batteries to be still in the esophagus. Which one of the following would be the most appropriate management?

A. Emergency endoscopic removal of the batteries.
B. Watch for the passage for 3 days.
C. Serial chest and abdominal X-rays.
D. Urgent laparotomy.
E. Reassure the parents.

A

A. Emergency endoscopic removal of the batteries.

Lithium batteries can cause liquefaction necrosis as rapidly as six hours. This occurs due to production of sodium hydroxide production caused by the current of the battery. If the battery is in the esophagus, it should be endoscopically removed as soon as possible. Esophageal burns and subsequent perforation occur adjacent to the negative battery pole (anode). Injury can continue even after endoscopic removal for days to weeks due to residual alkali or weakened tissue.

How well did you know this?
1
Not at all
2
3
4
5
Perfectly
5
Q

A 71-year-old woman underwent a hip replacement surgery 3 days ago. Tonight, you are called to visit her because she is unwell. She is agitated, cannot sleep and is exquisitely sensitive to noise. She has a history of anxiety and depression. Physical examination is unremarkable, and the vital signs are stable. An air blood gas (ABG) analysis reveals a PaO2 of 91 mmHg (normal: 80-95 mmHg). Which one of the following diagnoses best describes this clinical picture?

A. Urinary tract infection.
B. Pulmonary embolism.
C. Paroxetine withdrawal.
D. Benzodiazepine withdrawal.
E. Transient ischemic attack.

A

D. Benzodiazepine withdrawal.

Agitation, sleeplessness, and more importantly exquisite sensitivity to noise suggests benzodiazepine withdrawal as the most likely explanation to this presentation. History of anxiety and depression endorse this diagnosis as well, because it is common for patients suffering from anxiety to be on benzodiazepines as a means of relief.

The benzodiazepine withdrawal syndrome is highly variable. Common symptoms include insomnia, irritability, palpitations and sensory disturbances. Abrupt discontinuation in patients on high doses, e.g. greater than diazepam 50mg daily or equivalent, may result in seizures.

Option A: Urinary tract infection (UTI) would have presented with more pronounced symptoms such as fever, frequency, urgency and/or dysuria. Without these, and considering the normal exam results, UTI seems a remote possibility.

Option B: Considering the type of the surgery and subsequent immobilization, pulmonary embolism (PE) should be doubted and investigated; however, it is unusual for a patient with PE to have stable vital signs and normal physical exam findings, and more importantly a normal arterial oxygen pressure. Additionally,sensory disturbances are not a feature seen in PE.

Option C: SSIR (e.g. paroxetine) withdrawal syndrome can cause dizziness, vertigo, headache, nausea and flu-like symptoms as well as anxiety, confusion, irritability, excessive dreaming and insomnia. Although paroxetine can be the cause as well, the presence of sensory disturbances makes benzodiazepine withdrawal more likely.

Option E: Without any focal neurological deficits in history and clinical findings, TIA is unlikely.

How well did you know this?
1
Not at all
2
3
4
5
Perfectly
6
Q

A 42-year-old man, who was involved in a motor vehicle accident, is being evaluated in the Emergency Department. On examination, he has stable vital signs and an oxygen saturation of 96% on room air; however, he has marked difficulty in breathing due to intractable chest wall pain, which is worse on inspiration. He only takes shallow breaths. On examination, multiple bruises and tender points are noted on his chest. A non-contrast CT scan of the chest reveals multiple bilateral rib fractures as well a small pneumothorax on the right side. Which one of the following options is the most appropriate immediate treatment to help him with breathing?

A. Intubation.
B. Morphine.
C. Chest strapping.
D. Needle aspiration of the pneumothorax.
E. Underwater-sealed chest drain.

A

B. Morphine.

Pain control has a fundamental role to play in the management of rib fractures to decrease chest wall splinting and alveolar collapse. Patients with pain due to rib fractures, such as this, try to minimize their chest wall motion by shallow breathing and avoiding coughing efforts. Adequate pain management improves tolerance for deep breathing and coughing that will result in enhanced lung volume and clearance of secretions. These will reduce the risk of lung collapse and pneumonia.

A number of inpatient strategies have been proposed to optimize pain control in patients with rib fractures, including:

  • Regional anesthesia - Regional anesthesia techniques available for the management of multiple rib fractures include continuous epidural infusion, paravertebral block, intrapleural infusion, and intercostal nerve block.
  • Intravenous narcotics - Intravenous narcotics e.g. morphine can provide rapid adequate pain control. Intravenous route is preferred over subcutaneous or intramuscular injections because of the rapid and predictable onset of action. Although there is no evidence to support use intravenous nonsteroidal anti-inflammatory drugs (e.g. ketorolac) for pain control in rib fracture, they can be used to supplement and reduce narcotic use. However, these agents are avoided in patients who have a significant bleeding risk (e.g. hemothorax, solid organ injury) and those with renal insufficiency or hypovolemia. Patient-controlled analgesia (PCA) is advocated for patients with rib fractures because of a more timely access to pain medication by the patients and a reduced risk for excessive sedation.
  • The size of the pneumothorax and O2 saturation in this patient, make pain caused by fractured ribs the only respiratory problem for now, and adequate pain control is the first priority forcomfortable and painless breathing. This can be achieved by measures described above. Of the options only morphine can be used for pain control.

Option A: Intubation would be the most appropriate option for patients with fail chest with contraindications or inadequate response to less invasive measures such as CPAP or BiPAP.

Option C: Chest strapping by decreasing chest wall movements can be somewhat useful but it decreases adequate ventilation which is in disagreement with the management objectives. It is not an appropriate treatment.

Option D: Needle aspiration is the most appropriate next step in patients with tension pneumothorax for immediate relief. This then is followed by chest tubes and continues underwater-sealed drainage.

Option E: Water-sealed chest drain must be inserted for treatment of the pneumothorax; however, the pneumothorax is small and has not compromised oxygenation yet. It is not a priority at this stage.

How well did you know this?
1
Not at all
2
3
4
5
Perfectly
7
Q

A 35-year-old man is brought to the Emergency Department after he sustained a car crash. On examination, he has a blood pressure of 80/50 mmHg, heart rate of 110 bpm, and respiratory rate of 19 breaths per minute. He is pale and diaphoretic. Bedside pulse oxymetry shows an oxygen saturation of 95% on room air. Which one of the following is the most appropriate next step in management?

A. Supplemental oxygen.
B. Chest X-ray (CXR).
C. CT scan.
D. Intravenous colloid fluids.
E. Blood transfusion.

A

A. Supplemental oxygen.

In all trauma patients (like other patients) the ABC (airway, breathing, and circulation) protocol should be followed in a sequential manner. There is no comment regarding airway compromise. He also does not appear to have any respiratory distress. In addition, the normal oxygen saturation of 95% (94-100%) excludes respiratory compromise - at least for now.

The patient, however, is in pre-shock state evident by a systolic blood pressure of less than 90 mmHg. Reflexive tachycardia is
confirmatory. Although tension pneumothorax and cardiac tamponade can cause shock in the absence of hemorrhage, there are no clues to such conditions in the scenario. These make hemorrhagic shock the most likely cause of hypotension and tachycardia.

In all trauma patients in hemorrhagic shock, oxygen supplementation is beneficial and should be considered as the very next best step in management even if oxygen saturation is within the normal limits but not maximum. Maximizing the blood oxygen pressure compensates, to some extent, the decreased tissue oxygenation due to diminished perfusion caused by hypovolemia.

Option B and C: CXR and other imaging modalities such as CT scan should be withheld until the patient is hemodynamically stable.

Option D: Replenishing the lost volume is the most essential step after adequate oxygenation is ensured. Classically, isotonic solutions such as normal saline or ringer is used. Preference of colloids over isotonic fluids is still a place of significant debate.

Option E: Blood transfusion is considered when initial fluid resuscitation using 2 liters of intravenous fluid fails to restore and maintain adequate perfusion.

How well did you know this?
1
Not at all
2
3
4
5
Perfectly
8
Q

A 27-year-old woman is rushed to the Emergency Department by paramedics after she sustained a motor vehicle accident. Upon arrival, she is unconscious, has a blood pressure of 80/60 mmHg, pulse of 120 bpm, and respiratory rate of 22 breaths per minute. Stridor is noted on phsical examination. The patient has an oxygen saturation of 88% on room air. Supplemental oxygen is provided immediately, intravenous fluids are started, and a portable cervical X-ray obtained that reveals a fractured and dislocated cervical vertebra. Which one of the following is the most appropriate step in management of her airway?

A. Cricothyroidotomy.
B. Tracheostomy.
C. Intubation.
D. CPAP.
E. Increasing the oxygen flow.

A

C. Intubation.

This patient has concomitant compromised airway and a critical spinal injury at C2 level. Unconsciousness and stridor together mean that this patient should have an airway secured. This could be achieved by endotracheal intubation as the most effective and convenient method. However, intubation could be associated with immediate respiratory paralysis if the cervical lesion is above the C3 level or delayed phrenic nerve paralysis from ascending edema of the spinal cord in lower cervical lesions.

According to Advanced Trauma Life Support (ATLS®) guidelines, orotracheal intubation is the preferred method of airway management for patients with traumatic cardiopulmonary arrest, even with evidence of spinal injury. Orotracheal intubation using rapid sequence intubation protocol is recommended for patients who are breathing but unconscious and in need for airway control or ventilatory support. In-line spinal stabilization should be maintained throughout the procedure to minimize the spinal columns movement and reduce the risk of causing or exacerbating a spinal cord injury.

Option A: Cricothyroidotomy is indicated when an airway is required immediately in a patient who is not a candidate for orotracheal or nasotracheal intubation. Severe facial trauma is the most common indication. It is absolutely contraindicated in children younger than 12 years due to the risk of consequent tracheal stenosis. Relative contraindications include:
* Airway obstruction distal enough to the cricoid membrane that a cricothyroidotomy would not provide a secure airway with which to ventilate the patient
* Presence of a SHORT neck, which includes Surgery (history or prior neck surgery), Hematoma, Obesity, Radiation (evidence of radiation therapy), or Trauma/burns, making it difficult to locate the patient’s anatomical landmarks or producing an increased risk of further complications
* Tumor, infection, or abscess at the site of incision Lack of operator experience with the procedure

Option B: Tracheostomy should be performed in the operating room and is indicated for patients who are planned for prolonged mechanical ventilation.

  • Tracheostomy may be considered in the following situations:
  • Congenital anomaly (e.g., laryngeal hypoplasia, vascular web)
  • Upper airway foreign body that cannot be dislodged with Heimlich and basic cardiac life support maneuvers
  • Supraglottic or glottic pathologic condition (e.g., infection, neoplasm, bilateral vocal cord paralysis)
  • Neck trauma that results in severe injury to the thyroid or cricoid cartilages, hyoid bone, or great vessels
  • Subcutaneous emphysema
  • Facial fractures that may lead to upper airway obstruction (e.g., comminuted fractures of the mid face and mandible)
  • Upper airway edema from trauma, burns, infection, or anaphylaxis
  • Prophylaxis (as in preparation for extensive head and neck procedures and the convalescent period)
  • Severe sleep apnea not amendable to continuous positive airway pressure devices or other less invasive surgery
  • Tracheostomy may also be performed to provide a long-term route for mechanical ventilation in cases of respiratory failure or to provide pulmonary toilet in the following cases:
  • Inadequate cough due to chronic pain or weakness Aspiration and the inability to handle secretions

Option D: CPAP cannot provide a secure airway and is not an appropriate option in this patient.

Option E: Increasing the oxygen flow while the airway is compromised is futile.

How well did you know this?
1
Not at all
2
3
4
5
Perfectly
9
Q

A 23-year-old man is brought to the emergency department by ambulance paramedics after he sustained a stab wound in the chest in a street fight. On examination, he has a knife stuck in the left hemithorax. The patient is awake, oriented and cooperative. His blood pressure is 110/65mmHg, heart rate 100 bpm and respiratory rate 18 breaths per minute. There are no raised neck or forehead veins. Which one of the following is the most appropriate next step in management?

A. Remove the knife int he emergencey department and place deep sutures.
B. Remove the knife in the emergency department after you obtained a chest X-ray (CXR).
C. Transfer the patient to the operating room (OR), remove the knife in the OR and insert chest tube.
D. Insert chest tube in the emergency department and then remove the knife.
E. Perform a needle thoracocentesis for immediate decompression and then transfer the patient to the OR.

A

C. Transfer the patient to the operating room (OR), remove the knife in the OR and insert chest tube.

As a rule, all impaled foreign bodies should be secured in place and not removed until the patient is in the operating room (OR). The rationale behind this approach is that the object could have damaged major blood vessels, which are temporarily blocked by the object. If the object is removed, the pressure over the vessels is released, and life-threatening, potentially uncontrollable hemorrhage could ensue.

Such patients should be taken to the OR for removal of the impaled object in a controlled environment where potential bleeding after removal can be promptly controlled. A chest tube should be left in place for successful drainage of blood leak or air in the pleural space. As many as 80% of patients with penetrating chest trauma have hemothorax, pneumothorax, or both.

Any options suggesting removal of the knife in any place other than the OR is incorrect.

Immediate needle aspiration is the most appropriate next step in patients with tension pneumothorax which is not the case here. Tension pneumothorax presents with hypotension and difficulty breathing as well as other findings such as diminished air entry into the affected side, tracheal deviation away from the site of the injury, and diminished breath sounds and hyper-resonance of the affected hemithorax.

How well did you know this?
1
Not at all
2
3
4
5
Perfectly
10
Q

A 27-year-old man is brought to the Emergency Department after he sustained a stab wound to the left side of his upper chest. On examination, blood pressure is 120/90 mmHg, heart rate 110, and respiratory rate 19 breaths per minute. Bedside pulse oxymetry shows an oxygen saturation of 91% on room air. On inspection, the trachea is deviated to the left and breathing appears labored. Auscultation reveals decreased air entry into the left lung. The left side of the chest is dull to percussion. Which one of the following is the most appropriate next step in management?

A. Give him supplemental oxygen.
B. Insert chest tube in the left hemithorax.
C. Thoracocentesis.
D. Intravenous fluids.
E. Take him the operating room (OR) immediately.

A

A. Give him supplemental oxygen.

This patient has labored breathing and decreased oxygen saturation (normal 94%-100%) in the setting of penetrating chest trauma. Air entry is also diminished on the injured side. These can be caused by either hemothorax or pneumothorax. With dullness over the affected lung, hemothorax is the diagnosis, as in pneumothorax the presence of air in the pleural space gives rise to hyper-resonance on percussion on the affected side.

Regardless of the etiology, however, this patient is in obvious respiratory compromise evident by decreased lung air entry, elevated respiratory rate, and decreased oxygen saturation (< 94%) in the range of mild hypoxia; therefore, the first action must be application of supplemental oxygen according to the ABC protocol (airway, breathing , and circulation). This then should be followed by immediate chest tube insertion for evacuation of the air/blood form the pleural cavity as the definite treatment.

Option B: Chest tube must be inserted for management of hemothorax after supplemental oxygen is given.

Option C: Needle thoracocentesis would be indicated for emergent decompression if there was tension pneumothorax. In tension pneumothorax, the excess air in the pleural space leads to increased pressure within the affected hemithorax resulting in compression of the mediastinum away from the site of the injury. The heart then twists around its superior-inferior axis and kinks veins and decrease venous blood return to the heart. This leads to hypotension and shock. As mentioned earlier, the mediastinal structures and trachea are pushed away from the site of the injury, unlike this patient whose trachea is shifted towards the side of the injury.

Option D: Intravenous fluids are not indicated for now as the patient has normal blood pressure and heart rate.

Option E: Pneumothorax or hemothorax alone can be treated in the Emergency Department with chest tube, and is not an indication for transferring the patient to the OR per se.

How well did you know this?
1
Not at all
2
3
4
5
Perfectly
11
Q

A 32-year-old man presents to the Emergency Department with a knife impaled in his back under the right scapula. On examination, he has blood pressure of 85/50 mmHg, heart rate of 110 bpm and respiratory rate of 22 breaths per minute. There is adequate air entry into both lungs, but the breathing is labored and the patient can say one sentence at a time. Pulse oxymetry shows an O2 saturation of 91% on room air.

After starting him on supplemental oxygen by face mask and intravenous fluids, which one of the following will be the next best step in management?

A. Urgent CT scan of the chest.
B. Intubation.
C. Cross match.
D. Removal of the knife under general anesthesia.
E. Chest tube.

A

C. Cross match.

Low blood pressure and tachycardia in this patient indicate internal hemorrhage. Although tension pneumothorax and cardiac tamponade can cause similar picture, there is no clue to such conditions in the scenario. Tension pneumothorax is associated with shifted trachea away from the site of the injury, hyper-resonance, and/or decreased chest wall movements on the affected side. Cardiac tamponade results in decreased diastolic flow to the heart, pulsus paradoxus, hypotension, and tachycardia. Both tension pneumothorax and cardiac tamponade can lead to raised neck and forehead veins; however, veins might be flat due to concomitant hemorrhage and hypovolemia.

This patient has another problem as well – the impaled knife. As a rule, any impaled foreign body should be secured in place and not removed until the patient is in the operating room (OR). The rationale to this approach is that the object could have damaged major blood vessels, which are temporarily sealed by the object. If the object is removed, the pressure over the vessels is released and life-threatening and potentially uncontrollable hemorrhage could occur. This would be a grave condition in this patient which is already hemorrhaging.

In this patient blood should be drawn for typing, cross matching, and reservation after initial resuscitation with oxygen and fluid. Blood should be available in the OR in case the repair is prolonged, and bleeding cannot be stop immediately.

Option A: In this patient, measures such as initial resuscitation, blood typing and cross match, and immediate transfer to the OR should not be delayed for imaging studies such as CXR or CT scan.

Option B: Intubation of this patient should be considered in the OR if general anesthesia is planned. The knife is then removed, and any possible damage repaired.

Option D: This patient must be transferred to the OR for removal of the impaled object in a controlled environment and under general anesthesia. However, blood group typing and cross matching should be performed before that.

Option E: A chest tube will be secured in the OR after the knife is removed and damages repaired because approximately 80% of patients with penetrating chest injuries have hemothorax, pneumothorax, or both.

How well did you know this?
1
Not at all
2
3
4
5
Perfectly
12
Q

A 32-year-old man is brought to the Emergency Department by ambulance after he sustained a house fire. On examination, the patient is conscious but in severe distress due to pain. He has voice hoarseness, burns around the neck, and burned nose hair. Which one of the following is the next best step in management?

A. Intubation.
B. Oxygen by face mask.
C. Intravenous morphine for analgesia.
D. Debridment of the burns and application of topical antibiotics.
E. Intravenous fluids.

A

B. Oxygen by face mask.

In victims of a close environment burns, smoke and heat inhalation is associated with significant risk of airway obstruction. Any patient with sooth in the mouth or nose or burns in the face or mouth or around the neck should be monitored very carefully for airway compromise due to edema. Burn victims should be intubated before edema results in complete airway obstruction.

This patient has hoarseness that indicates airway involvement, and intubation should be considered. In meanwhile he should receive supplemental oxygen as the most important next step in management while arrangements for intubation are undertaken.

All other options are considered as parts of management, but none takes precedent over oxygenation and intubation.

How well did you know this?
1
Not at all
2
3
4
5
Perfectly
13
Q

A 19-year-old man is brought to the Emergency Department after he sustained a stab wound to the right side of his chest. The knife is still in. On examination, his vital signs are within normal range and there is no respiratory distress; however, air entry to the right lung is diminished. A chest X-ray (CXR) reveals a 25% pneumothorax on the right side. Which one of the following is the most appropriate next step in management?

A. Admit and observe.
B. Perform serial chest X-rays.
C. Removal of the knife in the operating room under general anesthesia and chest tube insertion.
D. Removal of the knife in the operating room under general anesthesia.
E. Thoracotomy.

A

C. Removal of the knife in the operating room under general anesthesia and chest tube insertion.

As a rule, any impaled foreign body should be secured in place and not removed until the patient is in the operating room (OR). The rationale behind this approach is that the object could have damaged major blood vessels, which are temporarily tamponaded by the object. If the object is removed, the pressure over the vessels is released, and life-threatening, potentially uncontrollable hemorrhage could occur.

Treatment of this patient’s pneumothorax should be undertaken in the operating room by inserting a chest tube and water-sealed chest drainage after the knife is removed and any damage is repaired.

Option A: Admission and observation is incorrect because not only this patient requires emergency removal of the impaled knife in the OR, his traumatic pneumothorax should be treated with chest tube.

Option B: Admission and observation with serial chest X-rays is considered in selected patients with occult asymptomatic pneumothoraces. A 25% symptomatic pneumothorax needs definite treatment with chest tube insertion.

Option D: Knife removal should be followed by chest tube insertion for drainage of blood and post-op pleural secretions even in the absence of pneumothorax.

Option E: Thoracotomy may be indicated for acute or chronic conditions.

Acute indications include the following:
* Cardiac tamponade
* Acute hemodynamic deterioration/cardiac arrest in the trauma center
* Penetrating truncal trauma (resuscitative thoracotomy)
* Vascular injury at the thoracic outlet
* Loss of chest wall substance (traumatic thoracotomy)
* Massive air leak
* Endoscopic or radiographic evidence of significant tracheal or bronchial injury
* Endoscopic or radiographic evidence of esophageal injury
* Radiographic evidence of great vessel injury
* Mediastinal passage of a penetrating object
* Significant missile embolism to the heart or pulmonary artery
* Transcardiac placement of an inferior vena caval shunt for hepatic vascular wounds

Chronic indications for thoracotomy include the following:
* Non-evacuated clotted hemothorax
* Chronic traumatic diaphragmatic hernia
* Traumatic cardiac septal or valvular lesion
* Chronic traumatic thoracic aortic pseudoaneurysm
* Non-closing thoracic duct fistula
* Chronic (or neglected) post-traumatic empyema
* Infected intrapulmonary hematoma (e.g. traumatic lung abscess)
* Missed tracheal or bronchial injury
* Tracheoesophageal fistula
* Innominate artery/tracheal fistula
* Traumatic arterial/venous fistula

How well did you know this?
1
Not at all
2
3
4
5
Perfectly
14
Q

A 36-year-old man is rushed into the Emergency Department by paramedics after he sustained a head-on collision as an unrestrained driver. He has a hard collar and his head is fixed to a spine board. On a quick review, he is conscious and fully oriented, but in distress as he is not able to feel his arms and legs. There is no visible site of active bleeding or limb deformity. Chest is clear to auscultation, neck veins are not raised, and abdominal exam is inconclusive. His blood pressure is 90/40 mmHg, heart rate 50 bpm and respiratory rate 18 breath per minute. His oxygen saturation is 88% on room air. Sphincter tone is decreased and there is no sensation or deep tendon reflexes below the neck. He is immediately placed on supplemental oxygen via face mask. Which one of the following would be the most appropriate next step in management?

A. Placement in Trendelenburg position.
B. Intravenous colloid fluids.
C. Blood transfusion.
D. Adrenaline.
E. Atropine.

A

A. Placement in Trendelenburg position.

The scenario suggests neurogenic shock. The most common cause of shock in a trauma patient is hypovolemic shock even in the presence of obvious spinal injury. In comparison, neurogenic shock is much less common and when the condition exists it can mask the presentation of hypovolemic shock; therefore, it is imperative that hemorrhage, as the most likely cause of decreased blood pressure is excluded. Neurogenic shock is always a diagnosis of exclusion.

In this patient, no site of active bleeding is noted. There is also no limb deformity to point towards long bone (or pelvic) fractures as the source of bleeding and decreased blood pressure. Abdominal exam is unremarkable as well.

Tension pneumothorax and cardiac tamponade are other conditions that can lead to decreased blood pressure by impeding venous return to the heart. For these to exist, chest should be involved. There are no comments as to chest involvement in the scenario; moreover, breathing is normal, and no abnormalities are found on auscultation.

Although the possible sources of occult bleeding should be thoroughly investigated, with no clues pointing towards other causes of hypotension, especially hemorrhage and obstruction, the next possible cause to consider in this patient must be neurogenic shock.

Neurogenic shock is classically characterized by hypotension, bradycardia, and peripheral vasodilatation. Neurogenic shock is due to loss of sympathetic vascular tone and happens only after a significant proportion of the sympathetic nervous system has been damaged – as can occur with lesions above the 6th thoracic vertebra. In a quadriplegic patient, blood pressure normally ranges between 80/40mmHg to 100/60 mmHg and pulse rate is down to 50 bpm.

The first measure to consider in patients with neurogenic shock is close attention to airway, breathing and circulation (ABC). Hypoperfusion to an injured spine can be associated with poor outcomes. Oxygenation should be monitored carefully, and oxygen be given. It is recommended that patients with traumatic spinal injury receive 15L/m oxygen via a non-rebreathing mask.

Hypotension and bradycardia are common features of neurogenic shock. According to guidelines by Neurosurgical Society of Australia, the first measure to consider is placement of the patient in Trendelenburg position. In neurogenic shock, the main mechanism of hypotension is pooling of blood in the peripheral venules and small vein; therefore, such maneuver can correct hypotension by increased venous return to the heart.

This patient has hypoxia evident by an oxygen saturation of 88% (normal ≥95%); therefore, supplemental oxygen should begin. In patients with neurogenic shock, systolic blood pressure should be maintained above 90 mmHg. Placement of the patient in Trendelenburg position (if there are no contraindication e.g., head trauma) should be considered next.

Option B: Isotonic fluids (not colloids) should come next after oxygenation and placement in Trendelenburg position. Careful monitoring for volume overload is a ‘must’.

Option C: Blood transfusion is considered in hemorrhagic patients if there is minimal response to adequate fluid resuscitation using crystalloids (e.g. normal saline). Patients with neurogenic shock as their sole underlying cause of their hypotension do not require blood transfusion.

Option D: Vasopressors or inotropes such as dopamine, nor-adrenaline (norepinephrine) or phenylephrine are considered for patients with hypotension resistant to position or adequate volume resuscitation. Urinary output can be an appropriate guide (urinary output of < 0.5cc/kg/hour). Adrenaline is not used for such a purpose.

Option E: Atropine is reserved for patients with bradycardia of < 50bpm. As mentioned earlier, it is quite common for patients with neurogenic shock to have bradycardia down to 50bpm. Atropine is only indicated if there is severe bradycardia (< 40 bpm), or when there is significant hemodynamic instability caused by it. Atropine should also be considered in patients who are undergoing maneuvers that can induce bradycardia by vagal stimulation e.g., nasopharyngeal suction or intubation.

How well did you know this?
1
Not at all
2
3
4
5
Perfectly
15
Q

A 56-year-old man undergoes hip surgery. In the following 24 hours, he only passes 200 ml of urine. During the surgery he was given 1.5 liters of normal saline and 2 units of packed red cells. Which one of the following investigations is more likely to find the cause of the oliguria?

A. Administration of 1 liter of normal saline in 1 hour and monitoring the response.
B. CT scan of the abdomen and the pelvis.
C. Retrograde pyelogram.
D. DSMA scan.
E. Urine analysis.

A

A. Administration of 1 liter of normal saline in 1 hour and monitoring the response.

Oliguria, defined as a urine output less than 30 ml/hour (0.5ml/kg/hour) is a common problem in post-operative period. This can be a consequence of fluid and blood loss resulting in increased release of aldosterone and antidiuretic hormone (ADH) by the adrenal cortex and posterior pituitary, respectively, in the first 24 hours after the surgery. Oliguria should be temporary and not last more than 24 hours. A urinary output of less than 500 ml within the first 24 hours warrants prompt investigations.

Before the oliguria is attributed to diminished urine output, it is important to ensure that the patient is not in urinary retention. A full, tender bladder is often a typical presentation of urinary retention. If in doubt, a catheter should be passed to check the urine volume in the bladder. Ultrasound can be used as an alternative.

Oliguria is caused by either of the following:

  • Poor renal perfusion (pre-renal failure e.g., hypovolemia or pump failure)
  • Renal failure (e.g., acute tubular necrosis)
  • Renal tract obstruction (post-renal failure)

Most cases of post-operative oliguria are secondary to hypovolemia. This should always be considered first until proven otherwise. This holds true about this patient as well. More so because he had developed hypovolemia during the surgery requiring resuscitation with fluid and blood.

In such circumstance, the next best step would be fluid challenge. One liter of normal saline is administered, and urine output monitored closely. An increase in urine output in response to fluid administration is both therapeutic and diagnostic for oliguria due to hypovolemia.

With renal failure as the underlying cause, oliguria will not respond to fluid challenge. If that is the case, investigation should include urine and plasma osmolality and electrolytes as well as an ultrasound for assessment of urinary tract obstruction as a possible cause of renal failure.

Option B: CT scan is not useful for diagnosis of renal failure. If imaging is required, ultrasonography is the preferred option.

Option C: Retrograde pyelogram requires contrast media. If renal failure is a concern, any option suggesting contrast imaging is incorrect. Contrast media is nephrotoxic and a hazard in patients with suspected renal failure.

Option D: DSMA scan has no role in workup for oliguria or renal failure.

Option E: Urinalysis for urine specific gravity is a part of routine investigation in patients with suspected renal failure and is indicated if the patient does not respond to fluid resuscitation. Other tests are urine and serum osmolality and urea, creatinine, electrolytes.

How well did you know this?
1
Not at all
2
3
4
5
Perfectly
15
Q

A 30-year-old woman is brought into the Emergency Department after falling from a horse while riding. She has a hard collar on and her head is taped to a hard spine board. She is fully conscious and oriented but unable to move any of her limbs. She answers the question in a normal voice. On a quick review, no visible active bleeding is noted. Chest is clear to auscultation and abdominal exam is inconclusive. No limb deformity is noted either. She has a blood pressure of 85/40 mmHg, heart rate of 50 bpm, and oxygen saturation of 90% on room air. Which one of the following is the most appropriate next step in management?

A. Atropine.
B. Adrenaline.
C. Oxygen.
D. Trendelenburg position.
E. Intravenous colloid fluids.

A

C. Oxygen.

In all trauma patients, initial survey and management starts with the ABC protocol (airway, breathing, and circulation).

This patient is fully conscious and oriented and answers the questions in a normal voice; therefore, her airway is not compromised, and no further airway management is indicated - at least for now.

Breathing comes next. An oxygen saturation of 90% on room air indicates mild hypoxia and requires supplemental oxygen as the most important next step in management. Additionally, hypoxia in the face of cord injury can adversely affect neurologic outcome. Arterial oxygenation should be monitored and supplemented as needed.

No site of active bleeding or any other clues to hemorrhage as the cause hypotension is present in this patient. Although, thorough investigation is required to exclude overt or occult hemorrhage as the most common cause of shock in trauma patients, given the clinical findings on the quick review, neurogenic rather than hemorrhagic shock is more likely.

Patients with neurogenic shock often have bradycardia down to 50bpm and a blood pressure ranging from 80/40 mmHg to 100/60 mmHg.

Option A: Patients with neurogenic shock have bradycardia of down to 50 bpm. No specific therapy is required unless there is significant hemodynamic instability or severe bradycardia (< 40 bpm).

Option B: Inotrope medications such as dopamine, norepinephrine, or phenylephrine maybe indicated in patients with resistant hypotension.

Option D and E: This patient may be initially placed in Trendelenburg position (head down) to relocate the pooled blood in the peripheral venules and small veins to the heart. This results in an increase in cardiac output and blood pressure. Longer term management includes judicious administration of crystalloids (not colloids) such as Hartman’s solution or normal saline to maintain a blood pressure of 90-100 mmHg. This is critical as hypoperfusion of the injured spine is associated with neurological sequelae and poor outcomes. These, however, do not take precedence over oxygen supplementation as the most appropriate next step in this patient.

How well did you know this?
1
Not at all
2
3
4
5
Perfectly
16
Q

A 48-year-old man with history of heavy smoking and alcohol drinking had a laparotomy with oversewing of a perforated duodenal ulcer. 24 hours after the operation, he is confused and disorientated. Which one of the following is the most appropriate initial investigation?

A. Blood alcohol level.
B. Arterial blood gases (ABG).
C. Serum electrolytes.
D. Chest X-ray.
E. Liver function tests.

A

B. Arterial blood gases (ABG).

Post-operative confusion/disorientation is one of the serious post-operative complications. The most common cause of post-operative confusion is hypoxia that must be excluded or confirmed by an arterial blood gas (ABG) analysis as the most appropriate initial step in management.

Hypoxia is most likely due to hypoventilation as a result of sedative drugs and post-operative analgesics; however, other etiologies such as electrolyte disturbances, atelectasis, and sepsis with gram negative organisms should always been considered and excluded. Sepsis with gram negative organism can lead to acute respiratory distress syndrome (ARDS) and shock in a short time. Drugs or alcohol withdrawal is another cause to consider.

Option A: Delirium tremens (DT) or alcohol withdrawal can also be associated with post-operative delirium and confusion. However, the diagnosis is clinical and bases on the history of alcohol consumption. Blood alcohol level do not help with the diagnosis.

Option C: Electrolyte derangements (particularly hypo- and hypernatremia) are other common cause of post-op confusion. An electrolyte panel is an important part of investigation in patients with post-operative confusion after hypoxia is excluded.

Option D: Chest X-ray is indicated in patients with post-operative confusion after the cause is confirmed to be hypoxia by ABG. Pulmonary atelectasis or pneumonia may be evident on chest X-ray.

Option E: Without the history of liver disease, confusion is unlikely to have been caused by liver disease. Liver function tests are not among routine investigations in patients with post-operative confusion.

How well did you know this?
1
Not at all
2
3
4
5
Perfectly
17
Q

A 40-year-old woman underwent cholecystectomy. On the first post-operative day, she develops fever and shortness of breath. After chest physiotherapy which one of the following is the next best step in management?

A. Continuation of chest physiotherapy.
B. Commencement of antibiotics.
C. Postural drainage.
D. Supplemental oxygen.
E. Bronchoscopy.

A

D. Supplemental oxygen.

The clinical findings of shortness of breath and fever in the early post-operative period (within 24 hours) are suggestive of atelectasis. Management of post-operative atelectasis starts with chest physiotherapy, followed by supplementation of oxygen to achieve an oxygen saturation of above 90%.

Option A: Continuation of chest physiotherapy is an appropriate step in management of atelectasis; however, after initial physiotherapy, supplemental oxygen should be given to maintain adequate oxygen saturation.

Option B: Antibiotics are not indicated in treatment of atelectasis unless chest infection follows.

Option C: Postural drainage helps with clearing of the secretions. The patient is asked to rest on the non-affected side to let the secretions drain by gravity. This is an important step in management of patients with atelectasis after initial physiotherapy and oxygen supplementation.

Option E: Bronchoscopy would be indicated if continuous chest physiotherapy fails. Routine use of bronchoscopy for removing secretions is not recommended.

How well did you know this?
1
Not at all
2
3
4
5
Perfectly
18
Q

In the first 24 hours after elective cholecystectomy, a 50-year-old woman develops a fever of 38.2°C and pulse rate of 110 bpm. The respiratory rate is 22 breaths per minute. She is smoker and had no pre-operative problems except the biliary pain. Which one of the following can explain this presentation?

A. Wound infection.
B. Pulmonary embolism.
C. Pulmonary atelectasis.
D. Bile leakage.
E. Pneumonia.

A

C. Pulmonary atelectasis.

Based on the history and clinical findings, pulmonary atelectasis is the most likely cause of this presentation. Pulmonary atelectasis is a quite common cause of post-operative fever in the first 24 hours of surgery, particularly in smokers.

Postoperative atelectasis is usually caused by decreased compliance of lung tissue, impaired regional ventilation, retained airway secretions, and/or postoperative pain that interferes with spontaneous deep breathing and coughing. Operations in the upper abdomen are most prone to development of atelectasis.

Option A: Wound infection presents more delay around 7th day post operation. Clinical signs of infection such as erythema, local tenderness, and/or purulent secretions are often found on clinical examination.

Option B: Pulmonary embolism typically occurs around 5th day post-operation, and presents with pleuritic chest pain, tachycardia, and tachypnea. The patient may or may not be febrile. It is less likely that a patient develops pulmonary embolism within the first day of surgery.

Option D: Leakage of bile presents with general malaise, abdominal pain, low-grade fever, and mildly elevated liver function tests. Shortness of breath and tachycardia are not common features.

Option E: Pneumonia often complicates atelectasis at around 3rd day post-op. It is early for development of pneumonia.

How well did you know this?
1
Not at all
2
3
4
5
Perfectly
19
Q

Eighteen hours after an uneventful cholecystectomy on a 45-year-old woman, she develops a fever and a cough. A chest X-ray (CXR) is obtained that is shown in the following photograph. Which one of the following is the next best step in management?

A. Chest physiotherapy.
B. Oral antibiotics.
C. Intravenous antibiotics.
D. Intravenous steroids.
E. Morphine.

A

A. Chest physiotherapy.

Of significance on the CXR, is the loss of the right heart border shadow, indicating collection in the right middle lobe. The onset of symptoms within the first 24 hours post-operatively, in addition to the CXR findings is consistent with the diagnosis of atelectasis. Atelectasis is common in abdominal and trans-thoracic procedures. Mucus retention in airways leads to post-obstructive collapse of the distal airways, particularly in the lower parts of the lung. Pneumonia can complicate atelectasis.

Post-operative atelectasis is managed as follows:

  • Removal of impacted secretion by percussion and encouraging the patient to actively cough.
  • Passive postural drainage.

The most important step in management is chest physiotherapy, followed by oxygen supplementation. If the above measures failed, a catheter can be passed through, guided by bronchoscope, for more vigorous removal of the secretions.

Option B and C: Antibiotics are not indicated unless fever persists more than 48 hours, suggesting superimposed pulmonary infection (pneumonia).

Option D: Steroids have no role in management of atelectasis.

Option E: Morphine is not indicated for management of atelectasis. Furthermore, morphine can decrease respiratory drive and worsens the atelectasis.

How well did you know this?
1
Not at all
2
3
4
5
Perfectly
20
Q

A 60-year-old man develops fever, tachypnea and shortness of breath 8 hours after a surgical procedure. A Chest X-ray show atelectasis of the lower lobe of the right lung. He has an oxygen saturation of 89% on room air. The patient undergoes chest physiotherapy, deep breathing, and incentive spirometry, but there is no significant improvement. Which one of the following is the next best in management?

A. Antibiotics.
B. CT angiogram.
C. Sputum examination.
D. Continuous positive airway pressure (CPAP) while arranging for bronchoscopy.
E. Continuation of physiotherapy.

A

D. Continuous positive airway pressure (CPAP) while arranging for bronchoscopy.

Atelectasis is common among patients who have undergone surgery especially in the setting of general anesthesia and use of opiate analgesics.

The initial management of atelectasis is with chest physiotherapy, deep breathing, active coughing, and incentive spirometry, followed by administration of supplemental oxygen. At occasions, these measures fail resolve the problem, mostly due to blockage of respiratory tract with a large mucus plug. The treatment of choice in this situation is bronchoscopic removal of the plug. In the interim, the patient must be put on continuous positive airway pressure for improved oxygenation.

Option A: Antibiotics are not required unless chest infection is suspected.

Option B: CT angiogram would be indicated if pulmonary embolism (PE) was suspected. PE typically develops around 5 days post-operation. Although not impossible, PE is less likely of a diagnosis given the early onset of symptoms. PE presents with sudden-onset pleuritic chest pain, shortness of breath, tachypnea, and tachycardia often around day 5 post-operation. With a massive PE, hemodynamic instability develops.

Option C: Sputum examination might be indicated if chest infection is suspected. It is not a part of workup for atelectasis.

Option E: After stabilization of the patient, continuous physiotherapy is recommended for prevention of further atelectasis; however, at this acute setting, it is unlikely to immediately relieve the shortness of breath and hypoxia.

21
Q

On the sixth day after an uneventful appendectomy on a 23-year-old man due to gangrenous appendicitis, he develops a fever. On examination, he has a blood pressure of 135/90 mmHg, pulse rate of 94bpm, and a temperature of 39°C. A small fluctuating tender swelling is noted on the lateral side of the wound. Which one of the following would be the most appropriate next step in management?

A. Oral antibiotics for 14 days.
B. Ultrasonography of the wound area.
C. CT scan of the abdomen.
D. Incision of the wound and drainage.
E. Intravenous antibiotics for 3 days, followed by oral antibiotics.

A

D. Incision of the wound and drainage.

Wound infection and abscess formation are common post-operative complications of surgical procedures. Usually wound infections are expected on day 7 post-operation or thereabouts. Abscess formation usually takes longer; nonetheless these timings are inaccurate, and surgical complications can potentially occur at any time during post-operative period. Simple wound infections present with fever, erythema, and local tenderness.

Abscesses, on the other hand, are more indurated and fluctuating, and characteristically contain puss. Abscesses tend to be associated with higher fevers.

Superficial wounds infections can be managed by suture removal and no antibiotics. Wound infections with cellulitis alone but no fluctuance to indicate underlying abscess can be treated with a course of antibiotics without drainage. However, when the clinical findings suggest an abscess, incision and drainage is the mainstay of therapy.

The fluctuating tender swelling in this patient is highly likely to be an abscess. Moreover, he has systemic symptoms of infection. In such circumstance, the most appropriate management is incision of the abscess and drainage. He also needs intravenous antibiotics. The wound should be left open for daily sterile packing.

Option A: Oral antibiotics are insufficient for management of this wound infection. Such infections often require intravenous antibiotics after incision and drainage has been carried out.

Option B and C: Wound infection can often be differentiated from abscesses on clinical basis. If they cannot be differentiated, ultrasonography is the investigation of choice to distinguish them. CT scan is not the favored diagnostic tool for this purpose. Here, the clinical findings are sufficiently diagnostic for an abscess without need for further evaluation.

Option E: Antibiotics alone are not sufficient for management of wound abscesses but are used in conjunction with incision and drainage.

22
Q

While doing exercise, a 16-year-old girl fell from a machine in the gym and had her head hit against the floor. She did not lose consciousness and remembers the incident. On examination, she is fully oriented, and has no neurological deficits. However, there is a swelling over the left side of the scalp. Which one of the following would be the next best step in management?

A. CT scan of the head.
B. Skull X-ray.
C. Observation for 6 hours and discharge if no neurological derangement develops.
D. Admit her to the hospital and observe for 24 hours.
E. Reassure and discharge home.

A

C. Observation for 6 hours and discharge if no neurological derangement develops.

Management of head trauma depends on its severity. Clinically, head trauma is categorized as minor, moderate or severe. Clinical features of each type are shown in the following table.

Management of head trauma depends on the severity and is as follows:

Minor head injury
* Observation for up to 6 hours with examination of vital signs, GCS, pupils, and limbs every 30 minutes
* Patient may be discharged from the emergency department for home care after carers are instructed about alarm signs
* If there is any doubt as to whether there has been loss of consciousness or not, assume there has been and treat as for moderate head injury.
* Adequate analgesia.

Moderate Head Injury
* If, on the history from the carers and/or ambulance, the patient is not neurologically deteriorating, he/she may be observed in the emergency department for a period of up to 6 hours after trauma with neurological observations every 30 minutes(conscious state, PR, RR, BP, pupils and limb power).
* The patient may be discharged home if there is improvement to normal conscious state, no further vomiting occurs, and patient is able to tolerate oral fluids.
* A persistent headache, large hematoma or possible penetrating wound may need further investigation and discussion with a consultant.
* Adequate analgesia.
* Consider anti-emetics - consider a longer period of observation if anti-emetics are given.

NOTE - The is no clear guidelines as to the exact time of observation of a patient with head trauma at the hospital.

Severe head injury
The initial aim of management of a patient with a serious head injury is prevention of secondary brain damage. The key steps are to maintain oxygenation, ventilation, and circulation, and to avoid rises in intracranial pressure (ICP). Management includes:
* Urgent CT of head and c-spine. Ensure early neurosurgical and ICU intervention.
* Cervical spine immobilization should be maintained even if cervical spine imaging is normal.
* Intubation and ventilation if:
Unresponsive or not responding purposefully to pain GCS persistently < 8
Loss of protective laryngeal reflexes
Respiratory irregularity
* In consultation with the neurosurgeon consider measures to decrease intracranial pressure:
Head elevation by 20-30 degrees (after correction of shock) and head in midline position to help venous drainage.
Ventilate to a target PCO2 35mmHg (4-4.5 kPa)
Ensure adequate blood pressure with crystalloid infusion or inotropes (e.g. noradrenaline) if necessary.
Consider mannitol or hypertonic saline
Consider phenytoin loading dose (20 mg/kg over 20 minutes, intravenously).
Control seizures if they occur
Correction of hypoglycemia
Analgesia: sufficient analgesia should be administered by careful titration

The following conditions need emergency transfer to a tertiary medical facility (if not already done):
* All severe head injuries
* Deteriorating conscious level (especially motor response changes) Focal neurological signs
* Seizure without full recovery
* Definite or suspected penetrating injury
* Cerebrospinal fluid leak
* Care required beyond the comfort level of the hospital

With no loss of consciousness, no focal finding on neurological exam, but a swelling (probably hematoma), this patient probably has a moderate head injury and requires to be closely observed for 4 hours.

23
Q

An otherwise fit 57-year-old woman spikes a fever of 39°C five days after an appendectomy for acute appendicitis. On examination, there is a tender, erythematous and fluctuant swelling at the medial end of the wound. Which one of the following options would be the most appropriate next step in management?

A. Administer high dose, broad spectrum antibiotics, intravenously.
B. Return the patient to the OR for resuturing the wound.
C. Start the patient on oral antibiotics.
D. Make an incision over the swelling to allow free draining.
E. Remove the underlying suture material to prevent sinus formation.

A

D. Make an incision over the swelling to allow free draining.

The tender erythematous fluctuant swelling at the surgical site suggests abscess formation as a post-operative complication of the surgical wound. Once abscess is formed, incision and drainage are the most appropriate management to considered first. Intravenous antibiotics are indicated as an adjunctive therapy.

Wound infection and abscesses are common post-operative complications of surgical procedures. Often, wound infections are expected on the around the 7th day post-op. Abscess formation usually takes longer; nonetheless, these timings are inaccurate, and surgical complications can potentially occur at any time during the post-operative period. Simple wound infections present with surrounding cellulitis (erythema and local tenderness). Abscesses, on the other hand, are more indurated and fluctuating and characteristically contain puss, and tend to cause higher fevers.

Option A: Intravenous antibiotics are used after incision is made to let free drainage of the abscess. Intravenous antibiotics are necessary, particularly in this patient with systemic infection and high fever but are not sufficient if used alone. Wound infections with cellulitis but no fluctuance can be treated with a course of antibiotics without drainage. However, when clinical findings suggest an abscess, incision and drainage is the mainstay of therapy.

Option B: Opening the wound, irrigation and re-suturing is not an effective management for abscesses. The sutures are removed, wound debridement and irrigation is performed, but the wound is left open and packed.

Option C: Oral antibiotics are insufficient for management of this wound infection. Such infections often require intravenous antibiotics after incision and drainage has been carried out.

Option E: Removal of the underlying suture material to prevent sinus formation is important but does not treat the patient’s current condition.

24
Q

A nurse from the surgery ward calls you to visit a 65-year-old inward patient for agitation. When you arrive at the ward you are informed that she underwent cholecystectomy 48 hours ago. Her file shows that she received prophylactic amoxicillin prior to the surgery. On examination, she is confused and delirious, has a blood pressure of 135/87 mmHg, heart rate of 110, and temperature of 38.4°C. She is agitated and difficult to deal with. Which one of the following is the most appropriate next step in management?

A. Start her on ceftriaxone.
B. Give her haloperidol.
C. Intravenous fluids.
D. Chest X-ray.
E. ABG.

A

B. Give her haloperidol.

Delirium following a major surgery in the elderly is a common postoperative complication. Delirium and confusional state after surgery are most commonly caused by hypoxia. Hypoxia can be caused by the effect of hypoventilation due to anesthetics or analgesics, or simply by shallow breathing due to pain.

Fluid and electrolyte disturbances, hypoglycemia, or infections can also cause post-operative delirium. Alcohol withdrawal or delirium tremens are other important etiologies not to miss. Urinary retention or fecal impaction should be thought of as well.

Management of delirium is by identification of the underlying etiology and treating it. This woman is febrile, making infection a likely cause of her delirious state. Although prophylactic antibiotics decrease the risk of post-operative infections, they do not elimnated such risk.
A confused and delirious patient can be difficult to deal with at times, and pharmacological treatment should be considered for sedation before other diagnostic or therapeutic measures are carried out. An ABG is often the very initial step to exclude hypoxemia as the most likely cause of delirium but attempts to obtain an arterial sample in an agitated delirious patient is not easy and may result in arterial injuries in a combative patient. An agitated patient may not let an oxygen face mask to be fixed in place. Under such conditions, medications are used to sedate and calm the patient.

Haloperidol is a convenient drug, which is most frequently used for this purpose and is the most appropriate next step in management.

Atypical antipsychotics such as risperidone, olanzapine or ziprasidone are second-line choices that can be used as alternatives; however, in the presence of extrapyramidal symptoms, they should be use in preference to haloperidol. Intravenous diazepam (not an option) is the medication of choice if alcohol withdrawal or delirium tremens are suspected from the history and clinical findings.

NOTE - Pharmacological therapy should only be considered in a delirious person with severe behavioral disturbance and/or severe emotional disturbance if their behavior threatens their own safety or the safety of others, is likely to interfere with essential medical or nursing care, or if the disturbance is causing significant distress.

Option A: Although the patient is febrile, antibiotics should not be administered unless bacterial infections are suspected after initial investigations such as full blood count (FBC), chest X-ray, etc.

Option C: This patient has a normal blood pressure and does not seem to be in urgent need for fluid resuscitation. However, it may be considered if further studies indicate otherwise.

Option D and E: ABG is often the first-line investigation to exclude hypoxia as the most common cause of post-operative delirium/confusion. A chest X-ray is indicated for evaluation and workup of conditions such as atelectasis, pneumonia, or pulmonary embolism if the patient is found to be hypoxic/hypoxemic. Again, obtaining an arterial sample for ABG can be difficult in this patient and should be attempted once the patient is easier to deal with.

25
Q

John, 5 years of age, falls from a swing and injures his right leg. Accompanied by his parents, he is now in the Emergency Department. He has a 5 mm deep laceration in a 2x3 abrasion on his right leg. The wound is contaminated with soil. You disinfect the wound and ask his parents about his tetanus vaccination status. He has received 2 doses of DTPa at 2 and 4 months of age, but not the third dose at 6 months or thereafter. Which one of the following is the appropriate management plan for him regarding tetanus vaccination?

A. One dose of DTPa now and a booster dose after 2 months.
B. One dose of DTPa plus tetanus immunoglobulin (TIG) now.
C. One dose of dT plus TIG now.
D. One dose of DTPa now plus a 5-day course of topical antibiotic cream.
E. One dose of DTPa now plus a 5-day course of oral penicillin.

A

B. One dose of DTPa plus tetanus immunoglobulin (TIG) now.

This child has a dirty tetanus-prone wound. Since there is no straight and clear definition for a tetanus-prone wound, all wounds other than clean, minor cuts should be considered tetanus-prone, with emphasis on the fact that some certain types of wounds are more prone to growth of C. tetani (dirty wounds). Such wounds include:
* Bite wounds
* Compound open fractures
* Foreign-body containing wounds (wood splinters in particular)
* Wounds complicated by pyogenic infections
* Wounds with significant tissue damage such as contusions and burns
* Deep penetrating wounds
* Superficial wounds contaminated with feces, dirt, soil, manure and dust, especially if disinfection is delayed more than 4 hours
* Wounds older than 6 hours

These wound are considered as dirty wounds.

NOTE - Reimplantation of an avulsed tooth is also considered a tetanus-prone event. The reason is that minimal washing and cleaning of the tooth is mandatory to increase the likelihood of successful reimplantation.

Management of a wound for tetanus prophylaxis depends mainly on the history of prior tetanus vaccination and whether the wound is clean or dirty, and is outlined in the following table.

Tetanus toxoid is available in Australia only in combination with diphtheria, with or without other antigens such as pertussis, inactivated poliomyelitis, hepatitis B and hemophilus influenza type b(Hib).

While the acronym DTPa (capital letters) signifies child formulations of combined diphtheria, tetanus and acellular pertussis vaccines, dTpa is used for formulations containing significantly lesser amounts of diphtheria and acellular vaccines. Such formulations are used for adolescents and adults.

John has not completed his primary course of tetanus vaccination. On the other hand, his wound is contaminated with soil and is dirty. With an incomplete course of primary immunization and a dirty wound, he should receive TIG for passive and a tetanus-containing vaccine immediately. Since he is under the age of 10 years, the appropriate vaccine for him is a DTPa containing vaccine.

Option A: John needs an immediate dose of a DTPa vaccine now (as the missed third dose). He also needs another dose as booster at least 4 weeks (one month) later. Although,the vaccination program offered is correct, the absence of TIG as a very important and crucial step in this option makes it incorrect.

Option C: Although immediate vaccination and TIG are steps to take for management of the tetanus-prone wound in this child, dTpa is not the age-appropriate vaccine to use in this setting and DTPa should be used instead. dTpa is used for those older than 10 years of age but younger than 18 years.

Option D and E: Antibiotics are not required for management of tetanus, but may be used for prevention of other wound infections. The decision as to whether prophylactic antibiotic coverage is given is a clinical decision made on a patient-to-patient basis. However, since these two option do not offer TIG, they cannot be accepted as correct options.

26
Q

John, 15 years of age, is a patient of yours since he was born. Today, his mother has brought him to your office after his left hand was bitten by his dog. The bite happened when John accidentally stepped on the dog’s tail and provoked the animal. On examination, there is a puncture wound which slightly bleeds when you remove the bandage the mother put on at home. No associated bone or tendon injury is found. You check his for his immunization status and know that his primary and booster vaccines (a total of 5 doses) has been given with the last booster at the age of 4 years. You debride the wound, disinfect it, and put a non-adherent absorbent plaster as covering. You also prescribe a 5-day course of co-amoxiclave for prevention of infections. Since it is a deep bite wound, you decide to also be on the safe side and give him prophylaxis against tetanus. Which one of the following would be the best course of action for tetanus prophylaxis for him?

A. A booster dose with DTPa (child formulation of diphtheria, tetanus, and acellular pertussis).
B. Tetanus toxoid (TT).
C. Adult diphtheria and tetanus toxoids (dT).
D. TT plus tetanus immune globulin (TIG).
E. A dose of combination vaccine of diphtheria toxoid plus tetanus toxoid plus whole cell pertussis (DTPw).

A

C. Adult diphtheria and tetanus toxoids (dT).

Due to high rates of mortality associated with C. tetani infection, it is important that every person with a tetanus-prone wound be assessed for the need of post-exposure prophylaxis against tetanus.

There is no clear definition of a tetanus-prone injury, and every wound, other than clean minor cuts, is considered tetanus-prone. Of these wounds, the following are defined as dirty with higher potentials for growth of the germ, severe infection, and poorer outcomes:
* Bite wounds
* Compound open fractures
* Foreign-body containing wounds (wood splinters in particular)
* Wounds complicated by pyogenic infections
* Wounds with significant tissue damage such as contusions and burns
* Deep penetrating wounds
* Superficial wounds contaminated with feces, dirt, soil, manure and dust, especially if disinfection is delayed more than 4 hours
* Wounds older than 6 hours

These wound are considered as dirty wounds.

NOTE - Reimplantation of an avulsed tooth is also considered a tetanus-prone event. The reason is that minimal washing and cleaning of the tooth is mandatory to increase the likelihood of successful reimplantation.

Management of a wound for tetanus prophylaxis depends mainly on the history of prior tetanus vaccination and whether the wound is clean or dirty, and is outlined in the following table.

John has had a full course of tetanus vaccination on recommended schedule. His wound is a deep bite wound which by definition (above) is dirty. The decision as to the need for tetanus prophylaxis and how to do that is based on the time that has elapsed since his last booster. For dirty wounds in a person with complete primary vaccination and the last booster dose within the previous 5 years neither vaccine nor TIG is indicated; however, if more than 5 years has passed since the last tetanus containing vaccine, an immediate booster dose will be required. The preferred vaccine depends on the age:
* If ≤ 10 years the booster dose should be given using a DTPa vaccine. The acronym DTPa (capital letters) signifies child formulations of combined diphtheria, tetanus and acellular pertussis.
* If >10 years, dTpa is used. dTpa signifies formulations that contain significantly lesser amounts of diphtheria and acellular pertussis vaccines. Such formulations are used for adolescents and adults.

For John, who is older than 10 years, a single dose of dTpa (preferred but not an option) or a combination of diphtheria and tetanus toxoid vaccines (dT) is the most appropriate action to take.

Option A: DTPa is a child formulation tetanus vaccine (up to 10th birthday) and not used for older persons.

Option B and D: TT alone is not available in Australia and other age-appropriate vaccines should be considered. On the other hand, John does not need TIG.

Option E: Whole-cell pertussis vaccines are suspensions of the entire B. pertussis organisms that has been inactivated, usually with formalin. Whole-cell pertussis is available in combination with diphtheria and tetanus vaccines (DTPw). Immunization with vaccines containing whole- cell pertussis have been frequently associated with minor adverse reactions such as erythema and swelling at the site of injection, fever and agitation. DTPa replaced DTPw for booster doses in 1997, and for all doses from 1999; therefore, not an appropriate option for John.

27
Q

While working in a factory, a 26-year-old machinist sustains a hand injury with a cutting machine. The wound is deep with bone exposure and crushed surrounding tissue. The wound is irrigated with copious amounts of normal saline and dressed accordingly. The patient is arranged to have his wound repaired in the operating room. The surgeon advises intravenous antibiotics for prophylaxis and tetanus prevention. He had a full course of tetanus vaccination as a child and also received a booster dose 3 years ago. Which one of the following is the most appropriate option for him regarding tetanus prophylaxis?

A. Human tetanus immune globulin (TIG), 250 IU, via intramuscular injection.
B. A single dose of dT (diphtheria/ tetanus vaccine).
C. Tetanus prophylaxis is not indicated for this patient.
D. dTpa plus 250 IU of TIG.
E. A single dose of DTPa.

A

C. Tetanus prophylaxis is not indicated for this patient.

All wounds other than clean minor cuts should be considered tetanus-prone and promptly managed according to current recommendations.

The decision as to tetanus prophylaxis is based on two very important features:
1. Tetanus-immunization history
2. Dirty versus clean nature of the wound

In addition to being tetanus-prone, the following wounds are considered dirty:
* Bite wounds
* Compound open fractures
* Foreign-body containing wounds (wood splinters in particular)
* Wounds complicated by pyogenic infections
* Wounds with significant tissue damage such as contusions and burns
* Deep penetrating wounds
* Superficial wounds contaminated with soil, manure and dust, especially if disinfection is delayed more than 4 hours
* Wounds older than 6 hours

The following table summarizes the management plan for tetanus-prone wounds based on the immunization history and the nature of the wound.

This man has completed his schedule vaccines and his wound is deep and associated with crushed tissue both of which, by definition, make the wound a dirty one. For dirty wounds in a patient with complete vaccination, the decision relies on the time elapsed since the last booster dose. While those who have received their last booster dose more than 5 years ago need another immediate booster dose with a tetanus- containing vaccine, people with a booster vaccine within the past 5 years do not need further action for tetanus prophylaxis; therefore, no treatment for tetanus prevention is required for this patient.

Option A, B and D: TIG is only indicated, for passive protection against tetanus in conjunction with a tetanus-containing vaccine, in persons with no, incomplete or uncertain tetanus vaccination history, who have a dirty wound. With complete vaccination this man does not need TIG. This man does not require any form of tetanus vaccines either.

Option E: DTPa is a child formulation tetanus vaccine (up to 10th birthday) and not used for older persons. dTpa is the preferred formulation for children aged 10 to 18 years. For older individuals (≥19 years) dT (diphtheria and tetanus but no pertussis) is also an option. Tetanus toxoid alone (TT) is not currently available in Australia.

28
Q

A 50-year-old man presents to the Emergency Department with a laceration in his left lower leg caused by pickax while farming. The wound is 1.5 cm deep and is contaminated with soil. You debride the wound, disinfect it, and put a non-adherent bandage on it. On enquiry about tetanus vaccination, you realize that he had his tetanus vaccination completed. He also had a hand cut 5 weeks ago, for which he received a single dose of dT. Which one of the following is the appropriate action for management of this patient?

A. Start him on a 5-day course of oral penicillin for wound infection prophylaxis.
B. Give him250 IU of human tetanus immune globulin (TIG).
C. Give him one dose of tetanus toxoid (TT).
D. Arrange for follow-up to check the need for antibiotics and discharge home.
E. Give him one dose of dT.

A

D. Arrange for follow-up to check the need for antibiotics and discharge home.

Tetanus is an acute disease which is often fatal. The disease is caused by clostridium tetani (C. tetani), a non-capsulated gram-positive rod and the neurotoxin it produces. The neurotoxin acts on the central nervous system resulting in muscle rigidity and painful muscle spasms. The incubation period is 3 to 21 days with a median of 10 days; however, it could be as short as 1 day or as long as several months. Shorter incubation periods are seen with significantly contaminated wounds. In such instances. a more severe clinical course and a poorer prognosis would be expected.

The disease initially presents with spams of the masseter muscles, dysphagia, neck, shoulder and back muscles pain and stiffness that may progress to generalized muscle spasms. Reduced ventilation, apnea and/or laryngospasm can occur. The cause of death is respiratory failure in most patients. Deep vein thrombosis, pulmonary embolism, rhabdomyolysis, fractures, muscle rupture, hypotension, hypertension, and cardiac arrest are other possible complications.

Due to high rates of mortality associated with C. tetani infection, it is very important that every person with a tetanus-prone wound be assessed for the need for post-exposure prophylaxis against tetanus.

There is no clear definition for a tetanus-prone injury, and every wound other than clean minor cuts is considered tetanus-prone. Of these wounds, the following are defined as dirty with higher potentials for growth of the germ, severe infection, and poorer outcomes:
* Bite wounds
* Compound open fractures
* Foreign-body containing wounds (wood splinters in particular)
* Wounds complicated by pyogenic infections
* Wounds with significant tissue damage such as contusions and burns
* Deep penetrating wounds
* Superficial wounds contaminated with feces, dirt, soil, manure and dust, especially if disinfection is delayed more than 4 hours
* Wounds older than 6 hours

NOTE - Reimplantation of an avulsed tooth is also considered a tetanus-prone event. The reason is that minimal washing and cleaning of the tooth is mandatory to increase the likelihood of successful reimplantation.

Management principles of a tetanus-prone wound relies on the previous history of tetanus immunization of the injured person as well as whether the wound is clean or dirty.

Clean minor cuts are not categorized as tetanus-prone wounds and for these wounds TIG is unnecessary, independent of previous tetanus vaccination history.

Adults with injuries considered to be tetanus-prone (all wounds other than clean minor cuts) should receive a booster dose of dT if more than 5 years have passed since their last dose of tetanus-containing vaccine. dTpa can be used as an alternative for provision of additional protection against pertussis. In children < 10 years of age, this dose of vaccine should be given as DTPa or a DTPa-combination vaccine, consistent with the child’s vaccination history and the recommended schedule. If there is any doubt about the adequacy of previous tetanus immunization in a person who has a tetanus-prone wound, TIG must be given as soon as possible, as well as tetanus toxoid-containing vaccine to provide both immediate passive and active protection.

The recommended dose for TIG is 250 IU, through intramuscular injection as soon as practicable following the injury. The dose should be increased to 500 IU if more than 24 hours have passed since the injury.

NOTE - Individuals with significant humoral immune deficiency may not have developed or maintained adequate immunity to tetanus, despite vaccination. Such persons require TIG for tetanus-prone wounds.

This man’s tetanus vaccination is complete. He also has received a booster dose 5 weeks ago (less than 5 years); therefore, he does not need further tetanus prophylaxis either active (vaccine) or passive (TIG) despite the fact his has a tetanus-prone and dirty wound. However, follow- up is required for treatment of potential wound complications including other wound infections.

Option A: Antibiotic prophylaxis is not indicated for prevention of tetanus, but its use for prevention of other bacterial infections is matter of clinical judgement. The best action, as offered as an option, is a follow-up to check if there are any signs of wound infection before the decision as to commencement of antibiotics is made.

Option B: TIG is indicated in cases with unknown, uncertain, or incomplete tetanus immunization with dirty wounds.

Option C and E: Tetanus toxoid is available in Australia only in combination with diphtheria, with or without other antigens such as pertussis, inactivated poliomyelitis, hepatitis B and Hemophilus influenza type b(Hib). Tetanus toxoid (TT) alone is not an available option for this patient. Moreover, he does not need tetanus vaccine (dT or dTpa) or immunoglobulin.

29
Q

A 57-year-old farmer presents to the Emergency Department after he accidentally steped on a rake in the farm and injured his left foot. On examination, there are 3 puncture wounds on his left sole which are not actively bleeding but the edges of the entry wounds are contaminated with soil. He has had a full course of tetanus vaccination including several boosters with the last one being 6 years ago. After disinfecting the wounds and putting non-adherent bandage over them, which one of the following would be the best action regarding post-exposure prophylaxis of tetanus in this patient?

A. Another tetanus booster with DTPa.
B. No tetanus immunization is required.
C. Another tetanus booster with dT.
D. Tetanus toxoid in form of dT and tetanus immune globulin (TIG). E. TIG and a course of oral penicillin.

A

C. Another tetanus booster with dT.

In management of a tetanus-prone wound there are two important features that guide the prophylaxis plan. The first one is the immunization status of the patient, and the second is whether the wound is clean or dirty.

Since there is no straight and clear definition for a tetanus-prone wound, all wounds other than clean, minor cuts should be considered tetanus-prone, with emphasis on the fact that some certain types of wounds are more prone to growth of C. tetani. Such wounds include:
* Bite wounds
* Compound open fractures
* Foreign-body containing wounds (wood splinters in particular)
* Wounds complicated by pyogenic infections
* Wounds with significant tissue damage such as contusions and burns
* Deep penetrating wounds
* Superficial wounds contaminated with soil, manure and dust, especially if disinfection is delayed more than 4 hours
* Wounds older than 6 hours

NOTE - Reimplantation of an avulsed tooth is also considered a tetanus-prone event. The reason is that minimal washing and cleaning of the tooth is mandatory to increase the likelihood of successful reimplantation.

Management of tetanus-prone wounds is outlined in the following table.

This patient has a dirty wound (deep and soil-contaminated) and full course of tetanus vaccination with the last booster dose 6 years ago. Since more than 5 years has passed since his last booster dose, he needs another immediate booster dose with dT (preferred) or dTpa as the most appropriate management option. He does not need TIG.

Option A: DTPa is a child formulation and not age-appropriate for this patient.

Option B: This option was the correct one if less than 5 years would have elapsed since his last dose of tetanus-containing vaccine.

Option D and E: TIG as a means of passive immunization is not required in individuals who has complete primary vaccination against tetanus (such as this man); therefore, any options suggesting TIG is incorrect. Antibiotics, on the other hand, may be used for prevention of wound infections in addition to tetanus prophylaxis. The decision regarding prescribing antibiotics for other infections than tetanus should be individualized for every patient.

30
Q

A few hours after insertion of a permanent pacemaker for a 55-year-old man, he develops shortness of breath and pleuritic chest pain. On examination, he has a blood pressure of 80/50 mmHg, heart rate of 110 bpm, and respiratory rate of 25 cycles/min. He has an O2 saturation of 96% on room air and a jugular venous pressure (JVP) of 6 cm (normal< 3cm). On auscultation, lungs are clear but heart sound are distant and muffled. Which one of the following is the investigation of choice to consider as the next best in management?

A. Serum troponin level.
B. ECG.
C. Chest X-ray.
D. Echocardiography.
E. CTPA.

A

D. Echocardiography.

Permanent pacemaker insertion into the heart chambers is performed through venous access. Most commonly, subclavian or cephalic veins are used for venous access. Rarely, internal jugular vein is used.

The procedure is minimally invasive and carried out under local anesthesia. There are, however, complications associated with the procedure such as bleeding, infection, pneumothorax, hemothorax, myocardial perforation, etc.

With chest pain, raised JVP, clear lungs, and importantly the distant muffled heart sounds, pericardial tamponade probably due to myocardial perforation is the most likely diagnosis. Myocardial perforation during or after the procedure is often a benign self-limiting condition, but rarely it can result in the disastrous cardiac tamponade. Pneumothorax is a more common complication of the procedure compared with myocardial perforation; however, if the case was a tension pneumothorax, decreased respiratory sounds in the affected lung and a normal heart auscultation would be expected.

Although cardiac tamponade is a clinical diagnosis, two dimensional and Doppler echocardiography play major roles in the identification of pericardial effusion and assessing the hemodynamic significance. Typical finding on two-dimensional echocardiography are a moderate to large effusion and swinging of the heart within that effusion. Also, echocardiography can be used as a guide for pericardiocentesis, which is the immediate and most vital treatment for the condition.

Option A and B: Serum troponin level and ECG would have been considered if the if the scenario was suggestive of myocardial ischemia. The presentation is very typical for cardiac tamponade rather than ischemia.

Option C: Chest X-ray is considered if the clinical presentation suggests pneumothorax, or other lung pathologies are thought of. Of note, in case of tension pneumothorax, the very first step in management is always urgent decompression with needle aspiration. Any further investigation should be reserved until the patient is hemodynamically stable.

Option E: CTPA is used when pulmonary embolism (PE) is suspected. PE is associated with tachycardia, tachypnea, and pleuritic chest pain. A normal lung auscultation does not exclude PE as a diagnosis; however, distant muffled heart sounds is not an expected finding in PE. Moreover, this patient has normal O2 saturation that makes PE even a weaker possibility.

TOPIC REVIEW
Potential complications of permanent pacemaker insertion include:
* Access-related complications
* Pocket-related complications
* Lead-related complications
Read Page 297-298 for elaborated explanation.

31
Q

An 11-year-old boy is recovered from a beach in Northern Queensland and brought to the Emergency Department of the local hospital. Upon arrival he has severe pain in the right foot and leg. Jellyfish tentacles are noted stuck in the skin of the right sole on a whip-like manner, and there is surrounding erythema and slight swelling. Which one of the following options would be the most important step in management?

A. Intramuscular morphine.
B. Pour vinegar on the foot and tentacles.
C. Immerse the foot in hot water.
D. Administer antivenom.
E. Start intravenous antibiotics.

A

B. Pour vinegar on the foot and tentacles.

Of all marine envenomation incidents, Jellyfish sting is the most common in Australia. Stings are caused by simultaneous discharge of many hundreds to thousands of nematocytes that are the venom-contained microscopic capsules in the jellyfish tentacles.

Jellyfish sting often causes acute sharp pain and an inflammatory response at the site of the sting. Such response may vary from local redness and tenderness of different intensity, urticaria, and swelling. Skin necrosis and destruction can follow. In severe envenomation, caused by significant number of stings, cardiorespiratory collapse can occur.

Jellyfish stings can be caused by a variety of jellyfish species, most of which cause minor stings with insignificant and self-limiting presentations. In Australia, most serious jellyfish stings are caused by the following two types of tropical jellyfish:

Box jellyfish – the Australian Box jellyfish (Chironex fleckeri) has a large box-like body of up to 20x30 cm and many tentacles. Contact with these tentacles results is whip-like skin marks and immediate pain which is often severe. Stings with multiple tentacles can lead to significant envenomation and cardiopulmonary arrest.

Jellyfishes causing Irukandji syndrome – stings from approximately 10 species of jellyfish can cause this syndrome that presents with severe generalized and often cramping pain within 5 to 40 minutes (typically 20-30 minutes) of the sting, nausea and vomiting, perspiration, restlessness and a sense of impending doom, and shortness of breath. Hypertension and hypertensive cerebrovascular events, pulmonary edema and acute heart failure are other possible presentations.

When stings occur, there might be non-discharged nematocysts left in the skin. These often release more venom into the victim’s body and further envenomation upon handling; therefore, it is important to prevent nematocysts form discharging their content. Vinegar (acetic acid 4- 6%) can neutralize nematocysts discharge in some species such as Box jellyfish and many types of those jellyfishes causing Irukandji syndrome and is the most appropriate initial step if stings by such species occur. Stings in the tropical regions of Australia such as Northern Queensland is almost always caused by such species.

Vinegar can result in more discharge from non-discharged species such as bluebottle jelly fish; therefore, this measure should be considered for tropical regions only.

NOTE - Vinegar is only used to neutralize the non-discharged nematocysts and does not provide pain relief or counteract the effects of already-injected venom.

Option A and C: Pain relief is of paramount importance if there is pain. Measures considered for pain control in jellyfish sting victims depends on the pain severity. While immersion of the affected area in warm water is the only method required for some patients, some other may need more analgesia, e.g. by morphine. Immersion of the stung area (if possible) is a useful pain control method for jellyfish stings especially non-tropical species. However, pain control should be considered after neutralization of non-discharged nematocysts.

Option D: In Australia, antivenom is available for Box jellyfish and other multi-tentacled jellyfish species. It can be used, especially for suspected severe envenomation. However, it is not a priority compared to pouring or spraying vinegar on the sting site.

Option E: Antibiotics might be considered later on if an infection occurs at the sting site or somewhere else.

References:
* RACGP - AFP - Marine envenomations
* Australian Resuscitation Council Guidelines - Guideline 9.4.5 Envenomaton - Jellyfish Stings

32
Q

A 46-year-old man undergoes laparoscopic cholecystectomy due to recurrent episodes of cholecystitis in the setting of gallstones. After 12 hours of the surgery, he develops chest pain. In addition to a complete focused history and physical examination, which one of the following would you consider first for this patient?

A. Chest physiotherapy.
B. CTPA.
C. ECG.
D. Echocardiography.
E. Chest X-ray.

A

C. ECG.

Chest pain in the post-operative period is a well-known complication that can be related or non-related to the surgery. Usual causes of operation-related include:
* Atelectasis
* Pneumonia
* Myocardial infarction
* Pneumothorax
* Pulmonary embolism

Of these, myocardial infarction and pulmonary embolism are potentially fatal etiologies to consider and exclude first. Among patients 45 years of age or older undergoing noncardiac surgery, complications of cardiac death, myocardial infarction, heart failure, or ventricular tachycardia occur in up to 5 percent. Of these, perioperative myocardial infarction is the most common and should be ruled out as a possibility; therefore, an** ECG would be the most appropriate next investigation to consider for this patient.**

Option A: Chest physiotherapy is the initial treatment option in patients whose pain is due to pulmonary atelectasis. Pulmonary atelectasis presents with pleuritic chest pain and shortness of breath often within the first 24 hours of the operation. Pulmonary atelectasis can be a diagnosis in this patient; however, unless other more serious causes of chest pain are not excluded safely, and patient is proved to have pulmonary atelectasis, this could not be a priority in management.

Option B: CTPA is the modality of choice for patients with high probability of pulmonary embolism as the diagnosis. Although not a rule, pulmonary embolism does not develop so early in the post-operative period; nonetheless, even if pulmonary embolism is suspected in this patient, an ECG and probably a chest X-ray should be done first to exclude other causes of the chest pain.

Option D: Echocardiography might be considered later in patients who are suspected to have myocardial infarction is the cause of the chest pain based on ECG findings or positive cardiac markers such as troponin.

Option E: A chest X-ray could be a correct in patients with respiratory symptoms and an unclear diagnosis. Also, patients with suspected pulmonary embolism require a chest X-ray to exclude other conditions than pulmonary embolism such as pneumonia as the etiology. A clear chest X-ray is on the other hand, a requirement for V/Q scan to be a reliable modality if decided as an investigation for pulmonary embolism.

32
Q

A 27-year-old man is brought to the Emergency Department after he sustained a motor vehicle accident. He is unable to move his arms and legs, and seems to have cervical spine injury. On examination, he has a blood pressure of 90/60 mmHg and pulse rate of 50bpm. He is alert and is breathing spontaneously. There is no visible bleeding source or limb deformity. His chest has multiple bruises and is tender to touch over the bruises but is normal on auscultation. The abdomen is quite normal on exam with no tenderness, rebound, or guarding. Which one of the following would be the next best step in management?

A. Intravenous colloid solutions.
B. Atropine.
C. Trendelenburg position (head down).
D. Adrenaline.
E. Dopamine.

A

C. Trendelenburg position (head down).

To answer this question correctly, familiarity with the mechanisms responsible for hemodynamic instability in a traumatic patient, who also has spinal cord injury is necessary.

In any trauma patient, hypotension should be assumed to be caused by hypovolemia until proven otherwise, even in a patient with an overt spinal injury. Sources of bleeding must be aggressively sought and controlled. Neurogenic shock is rare in comparison.

According to the scenario, there is no visible source of bleeding. Furthermore, absence of limb deformity makes long bone fractures as a source of bleeding unlikely. Abdomen is soft and non-tender, making intra-abdominal hemorrhage less likely, at least for now.

Other possibilities to consider as a cause of decreased blood pressure in this patient are bleeding into the chest (e.g., hemothorax) or obstruction of venous return associated with tension pneumothorax or cardiac tamponade. For these to be the cause, chest should have been involved, as is in this patient, evident by multiple bruises over the chest wall. However, the patient is breathing spontaneously and there is no abnormality on chest auscultation.

Although tachycardia is often a significant feature seen in hemorrhagic or obstructive shock, the presence of bradycardia does not exclude these conditions, as concomitant neurogenic shock can prevent compensatory tachycardia in response to decreased blood pressure. In this patient, however, with no evidence pointing towards other causes of hypotension, especially hemorrhage and obstruction, attention should be turned to other cause justifying the hypotension –** the neurogenic shock**.

Spinal shock and neurogenic shock are two different conditions that are often confused. Spinal shock is caused by acute spinal cord injury. Spinal shock is not a true form of shock. It refers to the flaccid areflexia that may occur after spinal cord injury and may last hours to weeks. It may be thought of as ‘concussion’ of the spinal cord. Spinal shock resolves as the soft tissue swelling improves. Associate features are absent deep tendon reflexes, sensation, and flaccid paralysis below the level of spinal injury.

Neurogenic shock, on the other hand is classically characterized by hypotension, bradycardia, and peripheral vasodilatation. Neurogenic shock is due to loss of sympathetic vascular tone, and happens only after a significant proportion of the sympathetic nervous system has
been damaged, as can occur with lesions above the 6th thoracic vertebra.

In a quadriplegic patient, the blood pressure normally ranges between 80/40 mmHg to 100/60 mmHg and pulse rate is down to 50 bpm.

NOTE - Spinal shock and neurogenic shock often coexist in patients with high spinal injuries.

Clinical features of neurogenic shock include:
* Hypotension – due to massive vasodilation
* Bradycardia – due to unopposed parasympathetic stimulation
* Poikilothermia (unable to regulate temperature)

Unlike hemorrhagic and obstructive shocks, patients with neurogenic shock have warm and flushed extremities because the sympathetic tone is lost and blood pools in the extremities.

These features occur within 30 minutes of cord injury above the 6th thoracic vertebra and can last up to 6 weeks.

The treatment goal for neurogenic shock is adequate perfusion and oxygenation. High flow oxygen should be administered (15 lit/min via non-rebreathing mask). Lesions above C5 often require intubation.

According to guidelines published by Neurosurgical Society of Australia, placing the patient in Trendelenburg position (head down) is the most important next step for initial management of decreased blood pressure and bradycardia. Trendelenburg position increases the return of pooled blood in the periphery to the heart and increases cardiac output and blood pressure.

Until the neurogenic shock resolves, it is of great significance to maintaining a systolic blood pressure of 90-10 mmHg. Isotonic fluids such as Hartman’s solution or normal saline are solutions of choice but should be administered with great caution. Excessive fluid administration may cause pulmonary edema because these patients are at risk for the acute respiratory distress syndrome (ARDS).

If the above measures fail to establish adequate perfusion, addition of inotropes is indicated. Dopamine or norepinephrine is used for treatment of protracted hypotension and bradycardia caused by higher spinal lesions. In lower lesions, vasopressors such as phenylephrine can be used.

Option A: Judicious administration of isotonic fluids are indicated in some patients, but colloids are almost never indicated for management of neurogenic shock.

Option B: Atropine is reserved for patients with bradycardia of < 50 bpm. As mentioned earlier, it is quite common for patients with neurogenic shock to have bradycardia down to 50 bpm. Atropine is only indicated if there is severe bradycardia (< 40 bpm), or when there is significant hemodynamic instability caused by it. Atropine should also be considered in patients who are undergoing maneuvers that can induce bradycardia via vagal stimulation e.g. nasopharyngeal suction, intubation, etc.

Option D and E: Inotropic support with dopamine or adrenalin (epinephrine) is rarely used and should be reserved for patients who have decreased urinary output (< 0.5cc/kg/hour) despite adequate fluid resuscitation.

33
Q

During your night shift in a rural hospital in Northern Territory, you receive a 27-year-old man 30 minutes after being bitten by a brown snake in the left ankle. Upon arrival, he has no symptoms. On examination, you notice scratches over the left ankle, but no fang marks are present. The nearest tertiary hospital is 150 km away. Which one of the following is the most appropriate next step in management of this patient?

A. Discharge him home because he is asymptomatic and there is no fang mark indicating a bite.
B. Air-transfer him to the tertiary hospital.
C. Give him a vial of antivenom now and another if symptoms develop.
D. Application of a tourniquet above the bite site.
E. Give him a vial of antivenom once he starts developing the symptoms.

A

B. Air-transfer him to the tertiary hospital.

It is very important that every suspected or definite snake bite is taken very seriously and assumed that envenomation has occurred until proven otherwise.

Unless massively envenomated, especially in children, where signs and symptoms may appear in a matter of minutes, life-threatening effects may not be apparent until hours later. Therefore, every patient with suspected or definite envenomation should be very carefully observed and monitored in a tertiary center with intensive care unit and access to anti-venom kits. Absence of visible bite marks does not reduce the importance of emergency care for this patient because there might be severe envenomation with absent or insignificant bite wound, fang marks, or other signs of a bite. Since a tertiary hospital is 150 km away, transferring the patient, preferably by air, is the most appropriate next step in management.

Option A: As mentioned earlier, there is often a time window between the envenomation and emergence of symptoms; hence, just because the patient is asymptomatic at present, he cannot be discharged. On the other hand, even with envenomation, snake bites may be painless and without visible marks. There might be no significant wound, local signs, or fang marks. In fact, in most cases there are no fang marks and often only a single mark or a scratch mark may be present (localized redness and bruising are uncommon in Australian snake bites)

Option D: Application of arterial tourniquet in snake bites is discouraged and not recommended.

Option C and E: Data suggests that antivenom might prevent certain envenoming syndromes if used early but may have little if any effect once major envenoming syndromes are established. There is no high-level evidence as a guide in this regard and clinicians have to balance the risks of giving antivenom (such as anaphylaxis and serum sickness) against the potential benefits of giving it early without waiting for confirmation of envenoming. The following are absolute and relative indications for antivenom administration:

Absolute indications:
* Reported sudden collapse, seizure or cardiac arrest
* Abnormal international normalized ratio (INR) (reference interval, 0.9–1.3)
* Any evidence of paralysis, with ptosis and/or ophthalmoplegia being the earliest signs

Relative indications:
* Systemic symptoms (vomiting, headache, abdominal pain, diarrhea)
* Leukocytosis
* Abnormal activated partial thromboplastin time (outside laboratory’s reference interval)
* Creatine kinase level > 1000 U/L

For now, this patient is asymptomatic and does not have a clinical (not laboratory) indication for antivenom; however, laboratory tests may show results such as an increased INR that mandate antivenom. Investigations for laboratory investigations to establish an absolute or relative indication for antivenom should not delay arrangements for transferring the patient to a tertiary center.

TOPIC REVIEW
Many of the snakes found in Australia are capable of lethal bites to humans. These include Taipans, Brown snakes, Tiger snakes, Death Adders, Black snakes, Copperhead snakes, Rough Scaled snakes, and many Sea snakes. Venom is produced in the modified salivary glands of the snake and forced out under pressure through paired fangs in the upper jaw. Snake venoms are complex mixtures of many toxic substances which can cause different effects in human victims. The life-threatening early effect in Australian snake bite is neurotoxic muscle paralysis which kills by causing breathing failure. Other significant effects include:
* bleeding due to coagulation failure
* muscle damage causing kidney failure headache
* nausea and vomiting
* occasionally, initial collapse or confusion followed by partial or complete recovery abdominal pain
* blurred or double vision, or droopy eyelids
* difficulty in speaking, swallowing, or breathing
* swollen tender glands in the groin or axilla of the bitten limb
* limb weakness or paralysis
* respiratory weakness or respiratory arrest

Important tips in initial management of snake bites:
* Keep the victim at rest, reassured and under constant observation
* Commence resuscitation if necessary
* Immobilize the affected limb
* Transport the victim to a medical facility, preferably by ambulance
* As a rule, all victims of suspected or definitive snake bites should be assumed to have been envenomated until proven otherwise.
* DO NOT cut or incise the bite
* DO NOT use an arterial tourniquet
* DO NOT wash or suck the bite because identification of venomous snakes can be made from venom present on clothing or the skin using a Venom Detection Kit.

NOTE - It is not recommended to kill the snake for purposes of identification, because medical services do not rely on visual identification of the snake species.

34
Q

A 35-year-old man presents to the Emergency Department after he sustained a bite by a drunk woman in a pub. The woman was a stranger who walked away after the incident. On history, he is fully immunized against tetanus with the last dose being 3 years ago. He is unaware of his hepatitis B immunization. On physical examination, there is a 2-cm bite wound 5 mm deep in the right arm that is actively bleeding and teeth marks around the wound. Which one of the following is the most appropriate next step in management of this patient?

A. Tetanus immunization.
B. Start him on zidovudine for HIV prophylaxis.
C. Give him hepatitis B vaccine and immunoglobulin.
D. Start him on a 5-day course of prophylactic azithromycin.
E. Suture the wound and review in 24-hours.

A

C. Give him hepatitis B vaccine and immunoglobulin.

Human bites have a higher complication and infection rate than animal bites. Although not true bites, clenched fist injuries are categorized under bite wounds. Clenched fist injury commonly presents as a small wound over the metacarpophalangeal joint of the dominant hand as a result of the patient striking another person’s teeth with a clenched fist. Human bite wounds to the hand more commonly develop bacterial infection than human bites at other sites, with clenched fist injuries conferring the highest risk, particularly because of the potential for breaching the metacarpophalangeal joint space to produce septic arthritis or osteomyelitis.

Management of human bites include the following:

STEP 1 - Wound care
The wound should be irrigated with normal saline and washed with water and soap. Gross foreign bodies should be removed and debridement considered if necessary. There is limited evidence regarding wound closure and suturing and decision should be made in a case- by-case bases; however, the following are general recommendations:
* Primary closure could be considered in carefully selected bite wounds where cosmesis is an issue.
* Primary closure of head and neck wounds with antibiotic prophylaxis: the risk of infection is only 1% due to enhanced blood supply and lack of dependent edema in these areas.
* Suturing is not recommended in wounds at high risk of infection.

NOTE - Patients with bites in the hand (either animal or human bite) should be referred early to hand surgeons to evaluate the need for exploration to prevent loss of function. Admission to hospital for intravenous antibiotic therapy may be required.

Elevation and immobilization is another important part of wound management to consider.

STEP 2 - Tetanus prophylaxis
All patients should be assessed for tetanus vaccination history and be given tetanus prophylaxis if necessary.

STEP 3 - Post-exposure prophylaxis for hepatitis B virus (and very rarely HIV virus)
The risk of transmission HIV infection following a human bite is extremely rare and practically negligible. Post-exposure prophylaxis for HIV is not routinely performed unless there is high index of suspicion that a biter’s saliva who is HIV positive has been mixed with his/he blood. There is no post-exposure prophylaxis for hepatitis C.

For hepatitis B, all patients, who has suffered a human bite and are not adequately immune to hepatitis B, or their immune status is unknown, it is recommended that they receive hepatitis B immunization with a full course of hepatitis B vaccine and hepatitis B immunoglobulin.

STEP 4 - Antibiotic prophylaxis
For otherwise healthy individuals, antibiotic therapy is usually not necessary for bites and clenched fist injuries with a low risk of infection (e.g. small wounds not involving tendons or joints that present within 8 hours and that can be adequately debrided and irrigated).

Antibiotic therapy is only necessary for bites and clenched fist injuries with a high risk of infection. These include:
* wounds with delayed presentation (8 hours or more)
* puncture wounds that cannot be debrided adequately
* wounds on the hand, feet, face or genitalia
* wounds involving deeper tissues (e.g. bones, joints, tendons)
* wounds in immunocompromised patients.

STEP 5 - Patient education and provision of reading materials
STEP 6 - Review in 24-48 hours

Of the given options, the most appropriate step for this patient would be giving him hepatitis B vaccine and immunoglobulin. However, if possible, a hepatitis B antibody titer to assess his immunity before commencement of immunization would have been the most appropriate option.

Option A: This patient has received a full course of tetanus vaccination with the last dose being 3 years ago; therefore, he does not require tetanus immunization.

Option B: Since the risk of transmission of HIV by human bites is practically negligible, prophylaxis with zidovudine is not necessary for this patient.

Option D: This patient does not have any of the aforementioned criteria for antibiotic prophylaxis and does not need it. Even so, azithromycin is not good choice.

Option E: As mentioned before, wound closure will be decided on a case-by-case basis and might be considered for this patient because the presentation is early and the wound is in an anatomical area where adequate debridement can be performed; however, only suturing the wound and review in 24 hours is not an appropriate management.

35
Q

A 32-year-old man is brought to the emergency department of a local hospital after he was recovered from a fire in a building. He was working in his office in that building when the fire started and was barricaded in the room for approximately 20 minutes before firemen were able to break into the room surrounded by fire and full of smoke. Which one of the following is the correct method of giving him oxygen?

A. 2L/min via nasal cannula.
B. 6L/min via Venturi mask.
C. 5-10L/min via simple face mask.
D. 10-15L/min via non-rebreathing mask.
E. Transferring him to hyperbaric oxygen chamber.

A

D. 10-15L/min via non-rebreathing mask.

In every victim of a fire especially in closed spaces, carbon monoxide poisoning should always be consider a possibility to investigate and treat immediately. Carbon monoxide (CO) is an extremely poisonous, odorless, tasteless and colorless gas, which is produced when there is incomplete combustion of carbon containing fuels (e.g. coal, petroleum, peat, natural gas, etc.). Inhalation is the most common route of exposure.

CO binds to hemoglobin with a much higher affinity than that of oxygen to form carboxyhemoglobin. CO-bound hemoglobin will not be able to transport oxygen anymore. This results in in tissue hypoxia. Moreover, CO combines with other hem compounds such as myoglobin and cytochrome oxidase.

Treatment of patients with CO poisoning depends on the severity of the toxicity, but for every patient with suspected CO poisoning high flow (10-15L/min) of normobaric oxygen via non-rebreathing mask should be started immediately. Intubation may be required in more severe cases.

NOTE - It is important to note that oxygen saturation by pulseoxymetry should not be used to guide oxygen therapy because pulseoxymeters cannot differentiate between CO-bound and O2-bound hemoglobin. So, any option suggesting that the need for and mode of oxygen therapy should be based on O2 saturation reading on pulseoxymeters would be incorrect.

Oxygen 2L/min via nasal cannula (prongs) (option A), 6L/min via Venturi mask (option B) or 5-10L/min via simple face mask (option C) will not provide adequate high flow oxygen to this patient and are not correct options.

Option E: Hyperbaric oxygen therapy using hyperbaric oxygen chambers are indicated for severely poisoned patients who are comatose, have seizures or other neurologic abnormalities, or have myocardial ischemia. Pregnancy is another indication for hyperbaric oxygen therapy. For such patients, hyperbaric oxygen therapy should be started as quickly as possible, ideally within 6 to 8 hours. There is no comments on symptoms or physical examination findings in this patient pointing towards the need for commencement of hyperbaric oxygen therapy. Moreover, even if he is in need for that, he must be started and maintained on high flow, normobaric oxygen while arrangements and preparations for hyperbaric oxygen therapy are made.

TOPIC REVIEW
Symptoms of CO poisoning depend on the extent of poisoning:

The main manifestations of carbon monoxide poisoning develop in the CNS and heart which are organs most dependent on oxygen. The symptoms are as follows:

Mild toxicity - throbbing temporal or frontal headache, fatigue, dyspnea on exertion, light-headedness and dizziness. In general, clinical symptoms of mild poisoning are non-specific and may mimic those of a non-specific viral illness, with vomiting, headache, malaise, weakness, fatigue and shortness of breath.

Moderate toxicity - severe headache, weakness, dizziness, nausea, vomiting, tachycardia, tachypnea, flushing, perspiration, decreased vigilance, diminished manual dexterity, impaired sensorimotor task performance, prolonged reaction time, difficulty thinking, impaired judgment, blurred or darkened vision, ataxia, loss of muscular control, tinnitus and drowsiness.

Severe toxicity - may produce syncope, seizures, confusion, disorientation, incontinence, ventricular dysrhythmias, cardiorespiratory depression, respiratory failure, coma and death. Delayed effects, attributable to hypoxia, usually result in neuropsychiatric effects.

Investigations:
Mild poisoning

  • Based on history and clinical picture.

Significant exposures

  • Determining carboxyhemoglobin (COHb) level when the patient is first seen and repeat every 2 to 4 hours until patient is asymptomatic or level is within the normal range.
  • ECG, EUC (electrolytes, urea, and creatinine), CK (creatine kinase), ABGs, and chest X-ray if symptomatic or if the COHb level is greater than 20%.

NOTE - COHb levels correlate poorly with signs and symptoms of toxicity. Interpretation may be confounded by delays in obtaining blood samples and therapeutic interventions such as oxygen administration.

  • CT scan or MRI if there are persistent neurological symptoms.
36
Q

While doing painting on a scaffold, John, a 48-year-old painter, falls off from a height of approximately three meters and lands on his both heels before he fell onto the ground. His coworkers called an ambulance to transfer him to hospital. In the emergency department, he denies any loss of consciousness, which is confirmed by one of his coworkers present at the scene who has accompanied him to the hospital. He is fully alert and responsive but in severe left foot pain and modest right foot discomfort as well as back pain. There are abrasions on the palms and a minor wound on the forehead. Vital signs are all within normal limits. There is severe tenderness, bruising and swelling below the left ankle and moderate tenderness over the lumbar area. Neurological examination is performed as far as feasible with unremarkable results. After giving him pain killers for pain, which one of the following would be the most appropriate next step in management?

A. CT scan of the head to exclude intracranial hemorrhage.
B. AP and lateral x-ray films of the left and right foot.
C. CT scan of the left foot.
D. Spinal x-ray series.
E. Immediate referral to orthopedic surgeon.

A

D. Spinal x-ray series.

Given the mechanism of injury (fall from height) and severe pain and bruising and swelling of the left foot, calcaneal fracture (fracture of the calcaneus bone of the heel) could be the most likely diagnosis.

Calcaneal fracture is uncommon and comprises only 1-2% of all fractures. Axial loading of the foot following a fall is the most common mechanism of injury. Calcaneal fractures are important from two different aspects. First is that this fracture is associated with long-term disability. The more important feature, however, is the fact that an axial load significant enough to cause a heel fracture can be associated with other injuries that may initially remain unnoticed, mostly due to distraction by severe foot pain. The patient might have sustained internal organs injury, spine injuries, and injuries of knees, hips and femur (mostly femoral head and neck). One study based on data from the American College of Surgeons National Trauma Data Bank (NTDB), showed that in patients with calcaneal fracture, 23% had spinal fractures, 18% head injuries, and 15% thoracic organ injuries. Most of the spinal fractures occurred in the lumbar spine.

Based on these facts, it is necessary that every patient with calcaneal fracture undergoes a thorough assessment for other possible injuries. Imaging studies should be ordered if any other injury is suspected in history and physical examination.

Given the commonality of spinal injury as an association with calcaneal fracture, and the presence of back pain, X-ray of the spine for assessment of any spinal fracture or injury is the most appropriate next step in management for this patient.

Option A: CT scan of the head would have been indicated if John had loss of consciousness or other signs or symptoms of an intracranial pathology. John is completely lucid and has not had any alteration in consciousness. He also have unremarkable neurologic examination; so, brain imaging is not indicated for now.

Option B: Plain films of the foot are the cornerstone of confirming the diagnosis. CT scan of the foot (Option C) is indicated in equivocal cases or where more details are required to decide about the management (extra-articular and intra-articular fracture are often managed differently). Although imaging of the calcaneus is necessary, more serious injuries must be excluded first.

Option E: Referral to orthopedic surgeon for definitive management of the calcaneus fracture is appropriate after other more serious injuries such as spinal injuries are ruled out.

37
Q

A 65-year-old man underwent an uneventful right carotid endarterectomy 4 days ago. Today, he presents with shortness of breath which is progressively becoming worse. Which one of the following would be the best immediate step in management?

A. Intubation.
B. Tracheostomy.
C. Opening all the wound layers in the Emergency Department.
D. Supplemental oxygen via face mask.
E. Opening all the wound layers in the operating theater.

A

C. Opening all the wound layers in the Emergency Department.

Carotid endarterectomy is the procedure of choice in some patients with carotid stenosis. This procedure is associated with complications, the most important of which are as follows:
* Perioperative stroke
* Myocardial infarction
* Hyper-reperfusion syndrome
* Cervical hematoma
* Nerve injury Infection
* Carotid restenosis

Postoperative cervical wound hematoma is a potentially lethal complication of carotid endarterectomy (CEA). The hematoma can rapidly distort the airway anatomy, making tracheal intubation difficult, and ultimately causing fatal airway obstruction. Post-CEA patients might develop airway emergencies related to hematoma formation either in the hospital or after discharge.

The progressive short of breast in this patient is most likely to have been caused by an expanding cervical hematoma. As always, management starts with the ABCD of life support (airway, breathing, circulation, and disability). The first step in this approach is ensuring and maintaining a patent airway. This patient has a rapidly deteriorating airway in need of urgent attention with intubation (option A). Initial method of airway management is based on the location, severity, and progression of the hematoma. Most patients can be managed successfully using a stepwise approach, beginning with awake fiber-optic intubation if time permits. If this fails, laryngoscopy, with either a direct or video laryngoscope, is a reliable backup. However, in a time-sensitive critical situation where there is impending respiratory compromise or airway loss, opening of all the wound layers at the spot (patient’s bedside in the Emergency Department in this scenario) to decompress the trachea and facilitate airway management should always be considered first as the most immediate action.

Surgical management of the airway using tracheostomy (option B) comes next if attempts to intubate patient fail.

Oxygen (option D) should be given after ensuring the patency of the airway or prior to intubation as a part of rapid sequence intubation. With an obstructed airway, supplementation with oxygen has minimal, if any, benefit.

Option E: Opening the wound layers in the operating theatre is the definite and most specific management of a patient who has a cervical hematoma and is clinically stable which is not the case in this scenario.

38
Q

A 19-year-old man is brought to the Emergency Department by ambulance after he sustained a gunshot wound to his abdomen. On examination, he has normal blood pressure and pulse rate with no sign of hemodynamic instability. The entry wound is in the epigastrium. There is no exit wound. The abdomen is tender to palpation. On anterior-posterior and lateral abdominal X-ray films, the bullet is found to be lodged in the right psoas muscle. Which one of the following would be the most appropriate next step in management?

A. Emergency ultrasound scan.
B. Abdominal CT scan.
C. Close clinical observation.
D. Exploratory laparotomy.
E. Diagnostic peritoneal lavage (DPL).

A

B. Abdominal CT scan.

Gunshot wounds are destructive to tissue. Destructive and wounding capability of a bullet is directly related to its kinetic energy and the damage caused by:
* Passage of missile
* Secondary shock wave
* Cavitation
* Exponential increase in injury with increasing velocity and efficient energy transfer
* Fractures (even in the absence of direct impact)

In every patient with gunshot wound(s), the very first step attending to airway, breathing, and circulation as usual. Hemodynamically unstable patients should be resuscitated immediately and vigorously, and any visible bleeding should be controlled while urgent transfer to the operating room for emergency laparotomy is planned and arranged.

In a stable patient, the next step would be a careful neurovascular exam. A high index of suspicion should always be kept for compartment syndrome secondary to increased muscle edema, especially in wounds inflicted by higher-velocity bullets. All wounds must be examined and documented.

For stable patients, plain radiographs and tripe contrast CT scan should be performed:
* Plain radiography
A chest radiograph should be obtained on all patients to exclude penetration of the chest cavity, even when the wound entry and exit are in the abdomen. Clues to look for on chest X-rays are hemothorax or pneumothorax or irregularities of the cardiac silhouette, air under the diaphragm (indicative of peritoneal penetration.)

Abdominal radiographs in 2 views (anterior-posterior [AP], lateral) are also obtained on all patients with gunshot wounds to help determine missile trajectory and to account for retained missiles (i.e., bullets, shrapnel, and foreign bodies).
* CT scan
Multidetector CT (MDCT) scanners have revolutionized the role of CT scanning in abdominal gunshot wounds. Intravenous and oral contrast, and in cases with suspected colorectal injuries. Contrast matter visualizes hollow and solid viscus organs injuries. In stable patients, this imaging modality can be applied relatively fast and efficient as an excellent test for those who do not need urgent exploratory laparotomy.

This patient is currently stable and has had abdominal X-rays performed showing a bullet lodged in the right psoas muscle. In its trajectory, the bullet may have injured other internal organs such as the liver, bowel, diaphragm, etc. For him, an abdominal MT scan to assess such injuries is the most appropriate next step in management.

Option A: Ultrasonography is an invaluable study in initial assessment of trauma patients, including those with gunshot wounds to the abdomen who are clinically unstable. It can rapidly determine the presence or absence of blood in the pericardial and peritoneal spaces. Focused assessment with sonography for trauma (FAST) is used for the evaluation of penetrating abdominal trauma because of its speed, noninvasiveness, and reproducibility in diagnosing intraperitoneal injury that requires laparotomy. FAST uses 4 views of the chest and the abdomen that are pericardial, right upper quadrant, left upper quadrant, and pelvis to evaluate for pericardial fluid indicative of cardiac injury and for free peritoneal fluid. Free fluid in the abdomen can be a sign of hemorrhage secondary to liver or splenic laceration or, less commonly, of spillage secondary to hollow viscus injury. FAST cannot exclude intra-abdominal injury, especially injury to the diaphragm or hollow viscus organs. This patient is clinically stable and ultrasound is not likely to provide specific information for clinical decision making.

Option C: With a bullet lodged in the psoas muscle that may have injured to the liver, diaphragm, kidneys, etc. this patient requires exploratory laparotomy, and close clinical observation as the only approach is definitely incorrect.

Option D: Exploratory laparotomy is emergently indicated for patients with abdominal gunshot wounds (or penetrating abdominal traumas in general) who are unstable, or for the stable patients who, through imaging studies, are found to have internal organ injuries such as perforations, lacerations, etc. This patient may be planned for exploratory laparotomy if CT scan shows injuries requiring surgical repair.

Option E: The role of diagnostic peritoneal lavage (DPL) in the evaluation of abdominal gunshot wounds is extremely limited. Latest generation multi-detector CT scanners (MDCT) provide high-resolution images with great speed, and have made DPL almost obsolete. DPL is only reserved for special circumstances – for example, assessment of a stable patient in a hospital where no MDCT is available.

39
Q

A 56-year-old man was diagnosed with non-Hodgkin lymphoma (NHL) and started on chemotherapy. After two years and a half, he was planned for stem cell transplant following a relapse. Four days after transplant, he develops a fever of 38.0 °C and severe pain while defecating. There is no blood covering or mixing with the stool. The stool is of normal consistency. Blood tests show a white cell count of
0.1x109/L (4-11x109/L), platelet count of 30x109/L (150-400x109/L), and hemoglobin of 90 g/L (120-160 g/L). Which one of the following is the most appropriate next step in management?

A. Complete isolation and monitoring.
B. Start him on broad-spectrum antibiotics.
C. Arrange for blood cultures and chest X-ray and plan further steps based on the results.
D. Platelet transfusion.
E. Sterilization of the gut with special care for anus.

A

C. Arrange for blood cultures and chest X-ray and plan further steps based on the results.

Bone marrow destruction using radiation or chemotherapy, is a process in bone marrow transplant before new bone marrow is injected. It takes a while for the newly transplanted bone marrow to take over reproduction of blood cells; therefore, there will be deficiency of all three cells lines (red blood cells, white blood cell, and platelets) during this period. Low blood cell count (leukopenia) places the patient at risk of infections including opportunistic infections, while low platelet count (thrombocytopenia) poses a risk of bleeding.

Post-transplant fever is a common post-op complication of bone marrow transplant without infection. Such fever often develops 4-5 days after the transplant even in the absence of infection and resolves by days 5-6. However, since infections are quite common in post-transplant patients and are associated with high mortality, any post-transplant fever should be attributed to infection and thoroughly investigated until proven otherwise. This is so important that commencement of empirical broad-spectrum antibiotics (option B) could also be considered; however, recent studies recommend that fever on days 4-5 that often is cytokine-related rather than infectious, and antibiotics are better used if there is high suspicion of infection through investigations. For this patient, blood culture and chest X-ray to investigate a bacterial infection before starting antibiotics is the most appropraite option to consider first.

Option A: At day 4 post-transplant, this patient is already in isolation and monitored closely.

Option D: With a platelet count of 30x109/L, this patient has thrombocytopenia. Thrombocytopenia is the term applied when the peripheral blood platelet count falls below 150x109/L. The problem is common in practice, but generally it presents no major clinical problems. However, if the platelet count falls below 30x109/L, significant bleeding might occur. In this patient no obvious bleeding has happened yet. While bleeding in the presence of higher platelet of demands platelet transfusion, in the absence of active bleeding, platelet transfusion for this patient is not necessary at this stage.

Option E: Colorectal problems are not uncommon in post-bone marrow transplant patients and may be caused by direct effect of radiation or chemotherapy, graft-versus-host disease of infections. Although, painful defecation in this patient should be seen to and investigated, it is not the most important priority in order.

40
Q

A 25-year-old man is brought to the Emergence Department after he was retrieved from a car accident by the paramedic staff. He had a head-on collision at high speed. Upon arrival, he has stable vital signs, and initial surveys clear airway, breathing, and circulation. His left elbow is significantly swollen and deformed and exquisitely tender to touch. The left hand feels cold, clammy, and numb and is pale. Radial and ulnar pulses are absent. Which one of the following is the most appropriate next step in management?

A. Reduction in the Emergency Department.
B. X-ray of the elbow.
C. Arrangements for immediate referral to the operating theatre.
D. Simple analgesia.
E. Ulnar nerve decompression.

A

C. Arrangements for immediate referral to the operating theatre.

This patient has an exquisitely painful deformed and swollen elbow that suggests a dislocated elbow with or without fracture. Dislocations can cause neurovascular injuries that pose a significant risk to the affected limb. The deadliest risk is acute limb ischemia which presents with the following (the 6 Ps):
1. Pain – sudden and severe
2. Pallor – commonly mottled
3. Pulselessness – loss of peripheral pulses
4. Paresthesia – decrease in sensation or loss of sensation
5. Paralysis
6. Poikilothermia – coolness of the affected limb

Once acute limb ischemia is suspected the following steps should be taken immediately:

  • Limb elevation to the heart level
  • Loosening of any restrictive bandages or dressings
  • Notify the specialty registrar immediately without hesitation to make arrangements for urgent transfer to the operating theatre
  • Placing the patient nil by mouth until review.
  • Increasing the frequency of neurovascular assessment to every 15 minutes until review.
  • Adequate analgesia.

This patient has classic signs of acute limb ischemia and requires urgent management before any further complications follow. Of the options immediate arrangement for being transferred to the operating theatre is the most appropriate next step in management. In the theatre, any vascular damage is repaired first, followed by nerve salvation (e.g., ulnar nerve decompression (option E) if compromised) or repair. Fractures and/or dislocations are fixed last.

Option B: While gentle and very careful alignment of the fractured/ dislocated limb is one of the priorities in the Emergency Department, reduction is a decision to be made after careful assessment by the specialist. Blind reduction in the Emergency Department may render further neurovascular injury.

Option D: Plain X-rays are important to obtain and are a part of routine physical examination in fractures/ dislocation. However, they do not change the management plan in this scenario. This patient needs to be transferred to the operating theatre due to acute limb ischemia. X-rays can be obtained after arrangements are made or en route to or in the operating theatre.

Optio E: Although analgesia is of paramount importance for this patient, it does not take precedence over immediate arrangements for review and treatment by specialist. Moreover, the pain associated with such a significant trauma is very unlikely to be responsive to simple analgesia. In such instances, opiates are often used.

41
Q

Irukandji syndrome could potentially be caused by:

A. Wasp sting.
B. Bee sting.
C. Spider bite.
D. Jellyfish sting.
E. Snake bite.

A

D. Jellyfish sting.

Irukandji syndrome is a painful, potentially lethal condition caused by certain jellyfish from the Cubozoa class (box jellyfish) species. This species has a small body (size of a coin) and usually four tentacles. The sting is usually mild and often goes unnoticed by the victim; however, after 20-30 minutes of the sting, a surge of catecholamine resulting in tachycardia, hypertension, agitation and generalized severe pain and cramps especially in the abdomen and back. Hypotension and, pulmonary edema, and potentially life-threatening cardiac complications can follow.

Victims should be transferred to the hospital for:
* Analgesia (oral and IV)
* Investigation and treatment of cardiac effects (echocardiogram, troponin)
* Cardiac monitoring

42
Q

A 24-year-old man presents to the Emergency Department of a rural hospital after he was bitten by a brown snake 15 minutes ago. On physical examination, he has a laceration of 3 cm on the lateral side of his left lower leg, but no fang marks can be found. Apart from pain and tenderness of the affected area, the rest of clinical assessment is inconclusive. The nearest tertiary hospital is 150Km away. Which one of the following is the most appropriate next step in management?

A. Application of pressure bandage and immobilization.
B. Application of venous torniquet.
C. Administration of antivenom.
D. Transferring the patient to the tertiary hospital by ambulance.
E. Irrigation and dressing of the wound.

A

A. Application of pressure bandage and immobilization.

In all definite or suspected snake bites, envenomation must consider having occurred until proven otherwise. Neither absence of fang marks nor lack of envenomation symptoms is not against envenomation because firstly, in many envenomation cases, no fang/bite marks are found. Secondly, unless massive envenomation occurred there is a time window during which the victim is completely symptomatic.

All snake bite victims should be transferred to a tertiary hospital (option D) where appropriate facilities such as laboratory, intensive care unit, antivenom kits and experts are available. Arrangements for immediate transfer of the patient to the tertiary hospital 150 km away should be made as soon as possible. In the meanwhile, the most important first aid measure is application of pressure bandage with immobilization (PBI). The bandage needs to be broad (15 cm) and preferably elasticized, rather than the previously recommended crepe bandage. The bandage is applied first over the bite and then to cover the whole limb at a pressure similar to that used for a sprained ankle. Immobilization is essential; the limb and whole patient must remain immobilized from the time of the bite or as soon as applicable. The use of a PBI more than 4 hours after the bite is unlikely to be effective.

The PBI can be removed when the initial clinical and laboratory assessment shows no evidence of envenoming, and the patient is in a facility where antivenom is available. For patients with envenoming, the PBI can be removed after administration of antivenom. As there are reports of cases where envenoming appears to be delayed by early application of a PBI but becomes evident soon after its removal, careful observation of the patient in the hour after PBI removal is essential.

The following should ALWAYS be avoided:
* Application of torniquet (option B).
* Cleaning, washing, and dressing the wound before the patient is in a facility for definitive care and treatment (option E).
* Incision if the bite site.
* Administration of antivenom anywhere but the tertiary hospital (if indicated) (option C).

TOPIC REVIEW
Many of the snakes found in Australia are capable of lethal bites to humans. These include Taipans, Brown snakes, Tiger snakes, Death Adders, Black snakes, Copperhead snakes, Rough Scaled snakes, and many Sea snakes. Venom is produced in the modified salivary glands of the snake and forced out under pressure through paired fangs in the upper jaw. Snake venoms are complex mixtures of many toxic substances which can cause different effects in human victims. The life-threatening early effect in Australian snake bite is neurotoxic muscle paralysis which kills by causing breathing failure. Other significant effects include:
* bleeding due to coagulation failure
* muscle damage causing kidney failure headache
* nausea and vomiting
* occasionally, initial collapse or confusion followed by partial or complete recovery abdominal pain
* blurred or double vision, or droopy eyelids
* difficulty in speaking, swallowing, or breathing
* swollen tender glands in the groin or axilla of the bitten limb
* limb weakness or paralysis
* respiratory weakness or respiratory arrest

Important tips in initial management of snake bites:
* Keep the victim at rest, reassured and under constant observation
* Commence resuscitation if necessary
* Immobilize the affected limb
* Transport the victim to a medical facility, preferably by ambulance
* As a rule, all victims of suspected or definitive snake bites should be assumed to have been envenomated until proven otherwise.
* DO NOT cut or incise the bite
* DO NOT use an arterial tourniquet
* DO NOT wash or suck the bite because identification of venomous snakes can be made from venom present on clothing or the skin using a Venom Detection Kit.

43
Q

A 12-year-old schoolgirl is brought to the Emergency Department of a tertiary hospital after she collapsed at school. En route to the hospital, she was started on dextrose 5% drip at a rate of 60 ml/minute. On examination after arrival, she has blood pressure of 180/110 mmHg, pulse rate of 50 bpm and respiratory rate of 12 breaths per minute. She has an O2 saturation of 95% on room air. Neurological exam shows the presence of doll eye reflex is present. Which one of the following would be the next best step in management?

A. Arrange for emergency CT scan of the head.
B. Stop the dextrose drip and start her on saline.
C. Give intravenous steroids.
D. Intubate her and start mechanical ventilation.
E. Neurosurgical reference.

A

B. Stop the dextrose drip and start her on saline.

The findings of high blood pressure and bradycardia (Cushing reflex) point towards increased intracranial pressure (ICP) as the most likely cause. Cushing reflex (also the vasopressor response, Cushing effect, Cushing phenomenon and Cushing reaction) is a physiological nervous system response to ICP. Cushing triad is: (1) hypertension, (2) bradycardia and (3) irregular breathing e.g., Cheyne-Stoke. This triad may indicate imminent brain herniation.

Increased ICP is more underpinned by the presence of the ‘doll eye’ sign (movement of the eyes in the same direction as the head) signifying involvement of brainstem, probably due to increased intracranial pressure.

The raised ICP is highly likely to be compromised by dextrose drip which has already been inappropriately started for the patient. Dextrose is rapidly consumed by cells and the remaining free water shifts into the brain extravascular tissue, and results in worsening of the edema, swelling, and more rise in the ICP. For this reason, the dextrose drip should be replaced with a normal saline as the most important immediate management.

An unconscious patient is not able to maintain airway patency. Furthermore, there is significant risk of aspiration; therefore, the patient should be intubated (option D), but not as the first priority at this stage, considering the fact that the patient is breathing spontaneously and is not hypoxemic (O2 saturation 95%). The patient should then be taken for CT scan of the head (option A) for determination of the likely causes of her problem. Consultation with or referral to the neurosurgery specialist (option E) should be arranged.

Intravenous methylprednisolone (option C) has shown effective in spinal cord compressions and cases of increased ICP due to tumors and abscesses. If, after neuroimaging, the cause of ICP was found to be an abscess or a tumor, corticosteroids may be considered as a part of management plan.

44
Q

A 3-year-old child is brought by his parents to the Emergency Department (ED) after he swallowed two button magnets. The child was initially well upon arrival at the ED and did not have any lethargy, abdominal pain, nausea, or vomiting. The parents explain that the event was witnessed and that they had brought the child over as soon as they could. The child does not have any illnesses and is otherwise well. An initial plain abdominal film was requested with results as shown:

The child was placed on NPO (nil per mouth), and intravenous (IV) hydration was started. Twelve hours later, you repeat the x-ray and note the same findings as the initial x-ray. The patient has stable vital signs all throughout and is still asymptomatic. Which of the following is the most appropriate management?

A. Perform serial x-rays every 4 hours.
B. Give the child laxatives.
C. Discharge him, and advise the parents to come back if he develops symptoms.
D. Refer to surgery for laparotomy.
E. Refer to a specialist for endoscopy.

A

E. Refer to a specialist for endoscopy.

Children of all ages can ingest a foreign body, but the incidence is most common between the ages of 6 months to 3 years old, and in those with behavioral or developmental problems. The management of foreign body ingestion depends mainly on three factors:
1. the site of obstruction and
2. the physical properties of the object, including size, shape, and composition
3. the timing of ingestion.

Site of obstruction
Assessment of the site of obstruction can be determined by asking about the following symptoms:
Esophagus:

  • Dysphagia
  • Food refusal, drooling
  • Chest pain
  • Vomiting, hematemesis
  • Throat pain
  • Unexplained fever
  • Altered mental state.

Esophageal obstruction usually occurs at these 3 sites:

  • Cricopharyngeal sling at the thoracic inlet (between the clavicles on CXR) – 70% of cases
  • Mid-esophagus (at the junction of aortic arch and carina on CXR) – 15% of cases
  • Lower esophageal sphincter – 15% of cases

NOTE - Impaction at other sites in the esophagus suggests an underlying esophageal abnormality.

Stomach and lower GI tract:

  • Abdominal pain, abdominal distension
  • Vomiting, hematemesis
  • Hematochezia
  • Melena
  • Unexplained fever

NOTE - Coughing, choking, and dyspnea are considered emergencies and are best treated as foreign body inhalation rather than digestion.

Determination of risk
High-risk objects

  • button batteries: most are asymptomatic, but they may erode GI mucosa and lead to fatalities — NOTE: parents may be advised to offer honey to children older than 12 months of age with suspected button battery ingestion; continuing at regular intervals until reaching the hospital
  • large objects (>6 cm long and/or >2 cm wide): potential entrapment at the pylorus
  • superabsorbent polymers (e.g., desiccants)
  • magnet plus a metal object or >1 magnet ingestion
  • lead-based objects that fail to transit through the stomach
  • multi-component objects (e.g., toys with lights, motors, and batteries) as they may break apart

High-risk children

  • Pre-existing GI tract abnormalities (repaired or unrepaired congenital abnormalities, eosinophilic esophagitis)
  • Neuromuscular disease

Timing
The timing of the ingestion would also be helpful in determining whether the object has passed through the esophagus or the stomach. In general, it takes about 6 to 8 hours for food to pass through the stomach.

The next step after the initial assessment is to determine whether imaging is needed if the suspected object is radiopaque. The following algorithm provides the next steps: (see flow chart)

In this case, a witnessed ingestion of button batteries, PA chest, or abdominal x-ray is the correct initial step. A lateral- chest x-ray may be indicated as well. On the x-ray, batteries are seen past the esophagus and in the stomach. However, button batteries are high-risk objects; therefore, urgent referral to specialist services (e.g., ENT, surgery, or gastroenterology) for removal of the batteries would be the most appropriate next step in management.

Passage of a foreign body lodged at the lower esophageal sphincter may occur after several hours, and the stable patient with normal anatomy may be observed in the hospital, and allowed to eat, with a follow-up x-ray in 12-24 hours.

In this child with a high-risk foreign body that has not passed after 12 hours of observation and conservative management, longer waits and repeating x-rays (option A) is inappropriate.

Although he is well and asymptomatic now, button batteries still have a risk of mortality and require referral for removal. Therefore, discharging the child home and advising him to return if there are symptoms (option C) is incorrect as well.

Option B: Laxatives are never used in the treatment of foreign body removal. Medications such as glucagon 0.02-0.03 mg/kg IV (max 0.5 mg), benzodiazepines, and nifedipine have been reported to assist foreign body passage through the lower esophageal sphincter in adults. However, their role in children has not been established, and they are unlikely to be successful in children with anatomic abnormalities.

Option D: Laparotomy to remove foreign bodies is an unnecessarily invasive procedure for foreign body removal in most cases. Unless the child is having severe symptoms or is rapidly deteriorating, laparoscopic removal is the method of choice.

` NOTE - If the child were of older age, consider intentional foreign body ingestion. This may warrant an initial mental health review and possible referral to psychiatry for further evaluation.`

45
Q

Which one of the following is not seen in zygomatic bone fracture?

A. Horizontal diplopia.
B. Vertical diplopia.
C. Enophthalmos.
D. Unstable temporomandibular joint.
E. Inability to open the mouth.

Photo supposed to be in answers
A

D. Unstable temporomandibular joint.

Zygoma bone has a prominent and important position in the facial skeleton. It forms a significant portion of the floor and the lateral wall of the orbit, and forms a portion of the zygomatic arch (malar eminence) which plays a key role in the determination of facial morphology.

The fractures of zygoma are often complex, and may affect either if the following portions:
* Zygomaticofrontal suture (along the orbital rim – lateral orbital wall)
* Zygomaticomaxillary suture (medially)
* Zygomatic arch

Most zygomatic fractures give fractures of maxillary sinus, zygomatic arch, inferior orbital wall and lateral orbital wall. Clinical picture in zygomatic fractures can include:
* Pain (70% of patients)
* Binocular diplopia (30% of patients) – The diplopia can be predominantly vertical, horizontal or oblique depending on the part of the zygomatic complex which is mostly affected. Diplopia can be secondary to muscle entrapment, neuromuscular injury, or intramuscular haematoma.
* Periorbital and/or subconjunctival ecchymosis (50% of patients)
* Paresthesias in the distribution of the infraorbital, zygomaticofacial, or zygomaticotemporal nerves can occur.
* Enophthalmos – Significant malar depression can be seen with step defects at the infraorbital rim, frontozygomatic suture, and zygomatic buttress of the maxilla intraorally.
* Inferior displacement of the lateral canthal tendon is common.
* Proptosis may be present due to orbital oedema or haemorrhage. Acute orbital hematoma may cause vision compromise and should be managed appropriately.
* Difficulty opening mouth – Posterior displacement of the fracture fragment may impinge on movement of the mandible causing difficulty with mastication and mouth opening. In addition reflexive spasms of masseter muscle may contribute to difficulty mouth opening and mastication as well.
* Crepitus from subcutaneous emphysema or proptosis and visual loss from orbital emphysema may occur with forceful nose blowing. Patients should be cautioned against this.
* In some patients, ipsilateral epistaxis also is noted as a result of lacerated maxillary sinus mucosa.

NOTE - Although zygomatic fractures are not true orbital blowout fractures, entrapment of orbital contents, enophthalmos, and diplopia with eye movement restriction may occur because of the contributions of the zygomatic bone to the orbital floor. Lateral orbital rim is displaced in most of zygomatic complex fractures.

Temporomandibular joint (TMJ) is a synovial joint between the mandible and temporal bone. Zygomatic fractures does not affect the TMJ structure (e.g. instability), but mouth opening and chewing may be restricted through mechanisms other than TMJ instability or involvement.

46
Q

A 26-year-old man presents to the Emergency Department after he was hit in the face by a racket in a squash game. Based on clinical findings, diagnosis of orbital floor fracture (blow-out fracture) is made. Which one of the following is the most consistent finding suggesting blow-out fracture?

A. Diminished vision.
B. Subconjunctival hemorrhage.
C. Anesthesia of the cheek.
D. Inability to open the mouth.
E. Horizontal diplopia.

A

C. Anesthesia of the cheek.

Inferior orbital wall fracture is associated with the following features:
* Binocular vertical diplopia
* Subconjunctival hemorrhage
* Peri-orbital ecchymosis
* Anesthesia or paresthesia of lower eyelid, upper lip, and maxillary area due to infraorbital nerve injury
* Limited vertical movement
* Enophthalmos
* Proptosis if there is severe orbital edema may lead to proptosis -once the edema subsides (usually 1-2 weeks) enophthalmos may be seen.

Of the given options, anesthesia of the cheek is the most specific finding indicative or orbital floor fracture and injury to the infraorbital nerve.

Option A and B: Diminished vision and subconjunctival hemorrhage can be found in blow-out fracture, as well as many other conditions that have eye injury as a feature.

Option D: Inability to open the mouth is a finding in complex zygomatic fractures, particularly if the zygomatic arch is involved.

Option E: Binocular horizontal diplopia is seen in lateral orbital wall fractures.

47
Q

A 32-year-old man was involved in a motor vehicle accident and sustained multiple injuries including tibia and pelvis fractures. The patient is managed accordingly. On the second day after hospital admission, you are called by the ward nurse because he has developed acute confusion and shortness of breath. A chest X-ray shows bilateral lung infiltrates. Which one of the following is the most likely diagnosis?

A. Pulmonary embolism.
B. Fat embolism.
C. Air embolism.
D. Pneumonia.
E. Drug withdrawal.

A

B. Fat Embolism.

Considering the history of pelvic and tibial fractures, clinical, and chest X-ray findings, fat embolism is the most likely diagnosis.

Fat embolism is a common problem following trauma to or surgery of lower extremities. These conditions can result in fat escaping from bone marrow into circulation and embolus formation.

The symptoms of fat embolism syndrome often present 24 to 72 hours after the insult, and involve lungs, brain and skin, and results in a triad of respiratory, neurological, and cutaneous manifestations:
* Lung problems including rapid breathing, shortness of breath (dyspnea), and hypoxemia – respiratory signs are the earliest findings in fat embolism and virtually exist in all patients.
* Neurological problems often follow respiratory involvement, and include irritability, agitation, headache, confusion, seizures, or coma
* Petechial skin rash is caused by occlusion of small blood vessels with emboli. Upper torso is the most common site involved. Eyes may be affected as well. Petechial skin rash is seen in approximately 60% of patients.

Diagnosis is made based on the history and physical exam findings, as well as exclusion of other possible causes.

Investigations include chest x-rays, blood tests, and CT scan a, and cardiac assessment with ECG and echocardiography to exclude cardiac causes of the presentation.

Option A: Pulmonary embolism is the second most likely differential diagnosis; however, the history and chest X-ray findings are more consistent with acute respiratory distress syndrome caused by fat embolism.

Option C: Minor cases of air embolism are common and cause minimal or no symptoms. Severe cases are characterized by hemodynamic collapse and/or acute insufficiency of certain organs, including the lungs, brain, and spinal cord. Dyspnea is present in almost all patients and may be accompanied by substernal chest pain, a sense of impending doom, lightheadedness, or dizziness. Signs of air embolism include a gasp or cough when the bolus of air enters the pulmonary circulation, a sucking noise as air is sucked into the intravascular space, a mill wheel murmur (a churning sound heard throughout the entire cardiac cycle), tachypnea, tachycardia, hypotension, wheezing, crackles, respiratory failure, altered mental status, focal neurological findings, crepitus over superficial vessels, livedo reticularis, and bubbles within retinal arteries. Although air embolism can cause the presentation in the scenario, with tibial and pelvic fractures in the history fat embolism is more likely to be the diagnosis.

Option D: Hospital acquired pneumonia can be seen in patients after 48 hours of hospital admission. The time of symptoms onset, bilateral infiltrates and absence of fever, cough and sputum makes pneumonia a less likely diagnosis.

Option E: Drug withdrawal symptoms tend to occur earlier. Besides, dyspnea and chest X-ray findings are inconsistent with the condition.

48
Q

A 22-year-old man has sustained multiple injuries during a motor vehicle accident. On examination, you find a laceration in his scalp, blood in the urethral meatus, and a left-sided pneumothorax. He has a blood pressure of 90/60 mmHg, pulse rate of 110 bpm and respiratory rate of 26 breaths per minute. The breath sounds over the right hemithorax are diminished. Neck and forehead veins are not distended and the trachea is normal in position. Which one of the following would be of greatest importance as initial management of this patient?

A. Management of possible urethral injury suggested by the presence of blood in the meatus.
B. Management of the pneumothorax.
C. Management of the scalp laceration.
D. Checking the tetanus immunization status.
E. Evaluation for possible blunt abdominal injury.

A

B. Management of the pneumothorax.

Current guidelines suggest initial management of traumatic patients in a sequence of A (airway), B (breathing), and C (circulation).

The clinical findings suggest a pneumothorax in the right hemithorax as well as pre-shock indicated by low blood pressure and compensatory tachycardia. Pneumothorax can compromise breathing and result in death if not treated promptly by chest tube insertion.

Option A: Blood in the urethra indicates urogenital injuries and should be investigated after the patient is hemodynamically stabilized.

Option C: Scalp laceration is another step to take after intravenous fluid resuscitation has been started and chest tube inserted. This can prevent ongoing blood loss. In the meanwhile, bleeding can be controlled by applying pressure on the laceration if required.

Option D: Tetanus immunization, if indicated, is important once the patient is clinically stable.

Option E: Although copious bleeding from scalp laceration can result in hemorrhagic shock and justify the low blood pressure and tachycardia, possible abdominal injuries should not be overlooked, and prompt investigation must be undertaken.

NOTE - Given the pre-shock status of the patient. evident by the decreased blood pressure and the tachycardia, intravenous fluid resuscitation would come prior to correction of the pneumothorax; however, if the pneumothorax was of tension type, immediate relieve by needle thoracotomy (not an option) would take precedence.